Сохранен 536
https://2ch.hk/spc/res/167732.html
Домены arhivach.top и arhivach.site временно не функционируют! Используйте домен ARHIVACH.XYZ.
24 декабря Архивач восстановлен после серьёзной аварии. К сожалению, значительная часть сохранённых изображений и видео была потеряна. Подробности случившегося. Мы призываем всех неравнодушных помочь нам с восстановлением утраченного контента!

Тред тупых вопросов и умных ответов. #24

 Аноним 13/04/15 Пнд 03:55:16 #1 №167732 
14288865161370.gif
Тред вопросов о жизни, Вселенной и всем таком.

Спрашиваем то, за что в других местах выдают путёвку в биореактор. Здесь анонимные ученые мирового уровня критически рассмотрят любые гениальные идеи и нарисованные в Paint схемы.

Прошлый тред тут: https://2ch.hk/spc/res/163953.html
Аноним 13/04/15 Пнд 09:37:35 #2 №167750 
>>167732
Почему между инфракрасными галактиками и микроволновым фоном нет нихуя?
Аноним 13/04/15 Пнд 10:00:10 #3 №167756 
14289084108040.gif
>>167750
А что там между ними должно быть?
Аноним 13/04/15 Пнд 10:10:34 #4 №167761 
>>167756
Галактики в промежуточных длинах волн.
Вместо этого там какой-то no signal и просто чернота.
Аноним 13/04/15 Пнд 10:14:36 #5 №167762 
>>167756
инфракрасный фон и микроволновые галактики?
Аноним 13/04/15 Пнд 10:37:12 #6 №167764 
14289106328980.png
>>167762
>>167761
А, в этом смысле.
Вселенная была непрозрачна какое-то время, пока шла рекомбинация. Атомы водорода и гелия не были стабильны от них отрывались электроны и снова улавливались атомами и снова отрывались. Фотоны постоянно сталкивались с ядрами и электронами. Это как вглядываться в густой туман. Пока эта хуита не закончилась ничего наблюдать нельзя было, ну кроме реликтового излучения.
Аноним 13/04/15 Пнд 12:02:16 #7 №167770 
Анон, а как добывается в космосе кислород? Вот на МКС, к примеру. Как он образуется? Если посадить там кучу растений, деревьев то можно ли автономно существовать? То есть что я думаю, берем корабль, наполовину засаживаем его растительностью, и вуаля, мы независим от кислорода. Нет?
Аноним 13/04/15 Пнд 12:27:06 #8 №167771 
>>167770
На МКС пользуются электролизом воды (Water Recovery System в модуле Destiny, Электрон-ВМ и Воздух в Заре и т.п.) Еще есть химические регенераторы кислорода (ТГК/SFOG), примерно как в бомбоубежищах. (Вика, OGM и т.д.) Восстанавливают состав атмосферы, при этом тратятся сами, и надо возить новые. Все это многократно дублируется. И постоянно горит и ломается.
> Если посадить там кучу растений, деревьев то можно ли автономно существовать?
Нет, потому что автономности только по кислороду недостаточно, да и 100% цикл нигде и не получен толком. Автономности по многим микроэлементам вообще невозможно достичь, проще брать запас.

На самом деле, как показал опыт эксплуатации Мира и МКС, на станциях намного больше проблем с завозом жрачки и запчастей, чем с завозом кислорода.
Аноним 13/04/15 Пнд 12:40:24 #9 №167773 
>>167750
Есть.
Аноним 13/04/15 Пнд 13:05:23 #10 №167781 
>>167764
А нейтрино тех времен можно наблюдать?
Аноним 13/04/15 Пнд 13:10:23 #11 №167782 
>>167781
Осталось научится отличать ТЕ нейтрино от ЭТИХ.
Это как попробовать отличить один атом кислорода от другого и понять, он в одной и той же звезде родился, или в разных.
Аноним 13/04/15 Пнд 13:12:17 #12 №167783 
14289199376720.jpg
>>167770
>берем корабль, наполовину засаживаем его растительностью
Надо исскуственные растения создавать оптимизированные для этого дела. Разница будит как между космонавтами и рэндомными попаданцами на мкс. Либо повторять процесс технологиями, чтоб кароч как солнечная панель, но вырабатывает не электричество, а кислород и углерод, либо от реактора, кароч топливный элемент наоборот. Ща Маск зделает дешевые запуски и начнут экспериментировать, и бионаука как раз созрела уже для первых опытов.
Аноним 13/04/15 Пнд 13:22:27 #13 №167784 
>>167783
>и бионаука как раз созрела уже для первых опытов.
>Созрела для первых опытов
>Первых опытов

У меня от этого первооткрывателя хлорелла передохла со смеху.
Аноним 13/04/15 Пнд 13:33:32 #14 №167787 
>>167784
И че уже пытались сконструировать, или все животинку мучаете?
Аноним 13/04/15 Пнд 13:35:27 #15 №167789 
>>167787
Что сконструировать? Какую животинку? Ты наркоман что ли?
Аноним 13/04/15 Пнд 13:39:24 #16 №167790 
>>167789
Ясно-понятно. Сконструировать лист хотя бы.
Аноним 13/04/15 Пнд 13:42:18 #17 №167791 
>>167790
Что тебе ясно-понятно?
Нахуя конструировать то, что давно сконструировано, или ты из изобретателей велосипедов? Тебе словосочетание "генная инженерия" ничего не говорит?
Аноним 13/04/15 Пнд 13:53:16 #18 №167794 
>>167791
Пиздец просто, мнение сельского агронома. Вместо того чтобы зделать нужное они будут пихать в банку сорняки и кукарекать о принципиальной невозможности чего-то там.
Аноним 13/04/15 Пнд 13:55:33 #19 №167795 
>>167794
>зделать нужное
Русский сделай, реально нужное тебе.
>они будут пихать в банку сорняки
Ты из быдлятни к нам залетел, ребенок??


Аноним 13/04/15 Пнд 20:27:25 #20 №167870 
>>167732
Что есть время? Что конкретно им является? Взаимодействие материи с пространством? Хотет ответ.
Аноним 13/04/15 Пнд 20:57:31 #21 №167888 
>>167870
>Что есть время?
Фундаментальное свойство Вселенной. Оно ничем не определяется.
Аноним 13/04/15 Пнд 23:21:11 #22 №168066 
14289564710540.png
14289564710561.jpg
Как называется эта единственная очень яркая звезда на юго-западе? Скрин из Google Sky Map.
Аноним 13/04/15 Пнд 23:28:40 #23 №168073 
>>168066
Use stellarium, Luke!
И обрати внимание на звездные величины в нем - чем больше цифра, тем ярче.
Аноним 14/04/15 Втр 00:19:35 #24 №168163 
Зачем закрывая "Созвездие" порезали "Аресы", когда можно было просто сократить кол-во вариантов? Ведь в итоге всё равно дорогая ракета, близкая к тяжелому "Аресу", но съевшая деньги на перепил.
И лого охуенное было.
Или SLS дико упрощена по сравнению с А5?
Аноним 14/04/15 Втр 04:05:08 #25 №168180 
>>168163
Затем что проталкивали разные люди. Зарубила и то и другое администрация, а SLS впоследствии протащил конгресс, под предлогом что надо занять чем-то спецов. SLS это типичный неэффективный проект с туманным будущим, форсируемый сверху для сохранения рабочих мест в стране. Как и Ангара. Попил, как принято говорить. Среди индустрии отношение именно такое.
Аноним 14/04/15 Втр 10:05:25 #26 №168207 
>>167764
Благодарю.
Аноним 14/04/15 Втр 13:17:42 #27 №168220 
Поясните, как разрабатываются йобы в NASA? Интересует, какую часть работы делает само агенство, а какую - подрядчики. Они ведь не просто раздают ТЗ и управляют готовыми аппаратами.
Аноним 14/04/15 Втр 14:02:32 #28 №168234 
>>168073
Я так понял, это Процион. А почему Юпитер над ним такой огромный? Его на небе вообще не видно.
Аноним 14/04/15 Втр 18:10:59 #29 №168310 
Насколько сильно взаимодействие Плутона и Харона? Ну вот т.е. у нас Луна гораздо слабее влияет на Землю, но и при этом вызывает приливы и отливы. Может ли там быть, например, что на поверхности Плутона, когда над этой стороной "пролетает" Харон значительно изменяется значение g? Или мб гравитация вообще начинает "работать" вверх в точку их общего барицентра? Поясните вот за это все.
Аноним 14/04/15 Втр 19:17:42 #30 №168339 
>>168310
На одной стороне Плутона из-за этой хуйни гравитация меньше, но последствия не такие уж большие, как может показаться. На форму планеты это 100% оказывает воздействие, а вот приливного разогрева не порождает, т.к. Плутон-Харон находятся в приливном захвате.
Аноним 14/04/15 Втр 19:28:43 #31 №168340 
>>168220
> Они ведь не просто раздают ТЗ и управляют готовыми аппаратами.
В общем-то в этом их суть и заключается. Подрядчикам даются те работы, на которые они находятся, в основном это производство, и частично разработка. Например ступени Сатурна-5 делались тремя разными компаниями под техническим руководством NASA. Но некоторые вещи делает сама NASA, если есть соответствующие ресурсы. Испытания, обеспечение полетов, НИОКР всяких уникальных хуиток в НИИ типа Ames и т.п. Обеспечением еще и USAF часто занимается, особенно предоставляет РЛС и прочую подобную технику.

>>168310
>когда над этой стороной "пролетает" Харон
Он все время висит над одной точкой.
http://en.wikipedia.org/wiki/Tidal_locking
Аноним 14/04/15 Втр 21:59:55 #32 №168418 
Сколько дельты добавляется к дельте ракеты, если пускать с экватора?
Аноним 14/04/15 Втр 22:16:30 #33 №168424 
Я понимаю, что ЧД это штука жутко теоретическая, но раз ядро сжимается, вещество не может под давлением вступать в теромядерную реакцию? Правда если и может, то какой толк, за горизонт излучению не вырваться, разве что состав ядра будет меняться. Хм, в таком случае в сторону наиболее "компактных" изотопов? Или там вообще уже не элементы, уже сингулярность нахуй?
Аноним 14/04/15 Втр 22:19:07 #34 №168426 
14290391478620.png
14290391478671.jpg
>>168418
В простейшем случае:
> Линейная скорость вращения Земли (на экваторе) — 465,1013 м/с
соответственно в запуске с экватора на восток получишь примерно столько же по сравнению с запуском с полюса.

На практике потери зависят от конкретной миссии - наклонения, на которое пускаешь, целевой орбиты и т.п. Например на ГПО с Байконура приходится менять наклонение орбиты, ибо трасса взлета проложена так, чтобы не проходить над нервными соседями, а смена наклонения это дорогая операция. Пикрилейтед-1 - типичная схема вывода на ГПО с Байконура. Предлагались разные схемы типа выхода на ГСО с огибанием Луны (!) который теоретически экономит дельту при запуске с северных широт. Но все такие схемы обычно упираются в живучесть разгонных блоков, окупаемость простоя полезной нагрузки и т.п.
Аноним 14/04/15 Втр 22:43:30 #35 №168440 
14290406102300.png
Аноны, это что за хуита, абу опять шутки шутить изволил? Или это у меня только?
Аноним 14/04/15 Втр 22:52:45 #36 №168454 
14290411651800.png
>>168440
у мнея норм
Аноним 14/04/15 Втр 22:56:02 #37 №168457 
>>168454
Уже всё стало нормально. Наверное некролис шалил.
Аноним 14/04/15 Втр 23:36:10 #38 №168511 
>>168424
>но раз ядро сжимается, вещество не может под давлением вступать в теромядерную реакцию?
Ядро звезды при коллапсе имеет вещество под таким давлением, что электроны вдавливаются в протоны, какая уж термоядерная реакция. А в ЧД уже тю-тю сингулярность во все поля.
Аноним 15/04/15 Срд 00:53:59 #39 №168655 
Как я, тупой ленивый уебан, могу помочь освоению космоса? Может, какие-нибудь распределенные вычисления можно на свой пека повесить, али еще чего такое?
Аноним 15/04/15 Срд 00:58:18 #40 №168661 
>>168655
Есть охуенная хуйня Stardust@Home - по мне таки самая охуенная из охуенных космических.
Аноним 15/04/15 Срд 12:20:13 #41 №168830 
Почему для полёта на Меркурий нужно столько же энергии сколько для полёта до Плутона? Почему сам полёт до Меркурия длится немногим меньше, чем полёт до Плутона?
Аноним 15/04/15 Срд 12:41:05 #42 №168837 
>>168830
Средняя орбитальная скорость Меркурия 48км/с, Земли 30км/с, Плутона 5км/с. Делай выводы.

> Почему для полёта на Меркурий нужно столько же энергии сколько для полёта до Плутона?
Пролёта мимо, или выхода на орбиту? Может ты путаешь бюджеты ХС конкретных аппаратов? Ибо Новые Горизонты не выходит на орбиту Плутона, ему для этого надо было бы каким-то макаром погасить скорость в 14км/с. (а у Мессенджера 800м/с с лишним)

> Почему сам полёт до Меркурия длится немногим меньше, чем полёт до Плутона?
Если ты о гомановском переходе, то до Меркурия 3 месяца лёту - меньше даже чем до Венеры.
Остальное зависит от траектории полёта, гравитационных маневров по пути и т.п. Так-то можно хоть 500 лет в сторону Плутона по ITN до Сатурна/Юпитера лететь, зато с околонулевыми затратами между точками.
Аноним 15/04/15 Срд 12:47:37 #43 №168841 
Гайс, а после МКС что-нибудь по орбитальным станциям планируется? Или будут до 2028 продлевать, а потом до 2032? И что у роскосого с их идеей запилить станцию на основе допиленного к середине 20-х МЛМ с надувными плюшками?
Аноним 15/04/15 Срд 12:51:57 #44 №168843 
>>168841
>Гайс, а после МКС что-нибудь по орбитальным станциям планируется?
Nyet, только разговоры.
Аноним 15/04/15 Срд 21:47:29 #45 №169088 
>>168661
Это круто, но я хотел просто поставить что-то на комп, чтобы оно использовало свободные мощности для нужных вычислений. Неужели нет возможности заставить работать свои ДВА ЯДРА, ДВА ГИГА на пользу науке и космонавтике?
sageАноним 15/04/15 Срд 21:51:05 #46 №169089 
>>168841

четкие планы только у китая, остальные бредят вариантами, кооперацией, коммерциализацией и прочим пидорством
Аноним 15/04/15 Срд 23:30:20 #47 №169138 
>>169088
SETI@Home, или как-то так. Кароч, много, юзай Гугл.
Аноним 16/04/15 Чтв 02:18:17 #48 №169184 
14291398975010.jpg
Насколько реально сейчас ресурсы из космоса добывать?
Пару лет назад гугл с насай собирались золотой метеорит пригнать, но что-то не слышно об этом больше.
Углеводороды со спутников газовых гигантов когда нибудь станет рентабельно на Землю завозить?
Аноним 16/04/15 Чтв 02:31:17 #49 №169188 
>>169184
Я искренне надеюсь, что к моменту появления технологий, углеводороды не будут источником энергии.
Всему своё время. Пока слишком дорого. Из наиболее реалистичного - Гелий-3.

Лол, а вообще это в рашке были заявления о "добыче полезных ископаемых" с каменюк, как обычно, в 2020, вот это всё.
Аноним 16/04/15 Чтв 02:32:11 #50 №169191 
>>169184
>Пару лет назад гугл с насай собирались золотой метеорит пригнать
>золотой метеорит
What the fuck am I reading?
>Углеводороды со спутников газовых гигантов когда нибудь станет рентабельно на Землю завозить
Нет.
Аноним 16/04/15 Чтв 03:03:15 #51 №169204 
>>169191
>What the fuck am I reading?
http://www.reuters.com/article/2012/04/24/us-space-asteroid-mining-idUSBRE83N06U20120424
>>169188
>углеводороды не будут источником энергии.
ну, их не только для этого будут использовать.

Так что там, современные технологи позволят при необходимости или нет?
Аноним 16/04/15 Чтв 03:12:22 #52 №169211 
>>169204
>Tech billionaires bankroll gold rush to mine asteroids
Пиздец ты надмозг.
Gold rush - это "золотая лихорадка", имеющая переносный смысл.
>современные технологи позволят при необходимости или нет?
Трудно представить себе такую необходимость.
Аноним 16/04/15 Чтв 03:14:09 #53 №169212 
>>169211
>A 30-meter long (98-foot) asteroid can hold as much as $25 billion to $50 billion worth of platinum at today's prices, Diamandis said.
С платиной перепутал, не важно, я по памяти писал, потом статью нашел.
Аноним 16/04/15 Чтв 03:38:23 #54 №169225 
>>169212
>can hold
А может и не содержать.
>as much as $25 billion to $50 billion worth of platinum at today's prices
Автоматическая миссия на астероид будет стоить миллиарды. Плюс добыча, плюс доставка обратно, плюс парковка на орбите и спуск на Землю.
НАСА уже писало на эту тему, там несколько спецов утверждали, что нужно закинуть к астероиду автоматическую станцию в 12 тонн, там будут роботы и 3D принтер, они там все и добудут. https://www.nasa.gov/mission_pages/asteroids/news/asteroidmining.html
На самом деле это примерное исследование. и одна разработка этой АМС-завода выльется в такой прайс, что шаттлам и программе Аполлон и не снилось.
Аноним 16/04/15 Чтв 03:44:55 #55 №169227 
>>169225
>billion
и есть миллиард, так что может и окупиться.
>одна разработка этой АМС-завода выльется в такой прайс, что шаттлам и программе Аполлон и не снилось.
хотеть!
Аноним 16/04/15 Чтв 03:45:34 #56 №169229 
>>169227
>и есть миллиард
туплю, это даже 1012
Аноним 16/04/15 Чтв 03:46:04 #57 №169230 
>>169227

2,5 миллиарда долларов стоимость Curiosity. Всего навсего колесный трактор с небольшим количеством приборов. С билетом в один конец. А тут добыча, с обратной доставкой и эквипмента на 12 тонн. Ояебу будет цена.
Аноним 16/04/15 Чтв 03:51:00 #58 №169232 
>>169230
> Ояебу будет цена.
Это в любом случае лучше чем нести демократию во имя луны.
>2,5 миллиарда долларов
Пентагон столько за полтора дня тратит.
Аноним 16/04/15 Чтв 03:55:29 #59 №169234 
>>169232
>Пентагон столько за полтора дня тратит.
Осталось только заставить их поделиться, ага.
Аноним 16/04/15 Чтв 03:57:22 #60 №169235 
>>169232
Проблема только в том, что нанесение демократий их сейчас интересует куда больше. Думаю, человечество по-настоящему за космос не хватится, пока на горизонте не замаячит настоящая жопа. Если успеет.
Аноним 16/04/15 Чтв 03:59:03 #61 №169237 
14291459433100.jpg
>>169235
Аноним 16/04/15 Чтв 04:02:51 #62 №169238 
>>169237
Ай, проебём мы астероид. Все ж вон, боевиков обсмотрятся и ябуть друг друга в жопы на геополитической арене. Какие там астероиды.
Аноним 16/04/15 Чтв 08:10:54 #63 №169241 
>>168511
Хорошо, спасибо.
Аноним 16/04/15 Чтв 17:40:39 #64 №169328 
14291952391960.jpg
Не пойму как это работает, вселенная вокруг тебя сгибается или как вообще?
Аноним 16/04/15 Чтв 18:05:53 #65 №169338 
>>169328
А зачем ей сгибаться?
Аноним 16/04/15 Чтв 18:57:37 #66 №169362 DELETED
>>169328
Это никак не работает, на картинке хуита.
Аноним 16/04/15 Чтв 19:00:49 #67 №169365 
14292000494960.jpg
>>169328
Так в ЦЕРНе оказывается машину времени строили, вот оно чо.
Аноним 16/04/15 Чтв 19:33:09 #68 №169379 
Есть где-нибудь в инетернетах реконструкции внеземных небосводов? Допустим я хочу посмотреть на небо с меркурия (и не только), оценить размер венеры на небосводе. Посмотреть насколько большое солнце там. Ну ты понел.
Аноним 16/04/15 Чтв 19:39:27 #69 №169386 
14292023673930.jpg
>>169328
> как это работает
как работает что?
Вселенная работает как часы.
Черная дыра работает как обычно с 10 до 18
Принцип действия LHC основан на ускорении заряженных частиц магнитным полем.
Кротовая ныра работает как в интерстелларе, если вообще существует.
Машина времени не работает, потому как не существует.
Аноним 16/04/15 Чтв 19:41:54 #70 №169389 
14292025147000.png
>>169379
приклейтед, space angine. stellarium должен мочь в это, не проверял.
сага 16/04/15 Чтв 20:05:08 #71 №169408 
14292039083310.jpg
14292039083321.gif
>>169328
>как это работает
Как в звёздных вратах. Заходишь и полетел.
Аноним 16/04/15 Чтв 20:12:35 #72 №169414 
Кстати, насчет порталов. Планеты ведь двигаются относительно друг друга, а ещё звезды тоже на месте не сидят и голактеки тоже. Как сделать портал неподвижным относительно какой либо удаленной планеты? А то ты проходишь в портал открытый как ты думал на другой планете, выходишь в космосе планета-то улетела.
Аноним 16/04/15 Чтв 20:14:07 #73 №169417 
1. Возможно ли создание воздушно-реактивного двигателя для бескислородной атмосферы? ну т.е., чтоб роль окислителя выполнял забираемый из окружающего пространства воздух, не содержащий кислорода.
2. Возможно ли создание этого самого двигателя для работы в атмосфере какого-либо из имеющихся тел Солнечной системы кроме Земли?
Аноним 16/04/15 Чтв 20:17:16 #74 №169420 
>>169417
Забирать топливо из атмосферы, а окислитель везти с собой.
Аноним 16/04/15 Чтв 20:56:49 #75 №169433 
Вот интересно, двигатель забирающий топливо из атмосферы, а окислитель несущий в баке, будет устроен как обычный реактивный двигатель или он не сможет работать?
Аноним 16/04/15 Чтв 21:10:06 #76 №169440 
>>169338
Потому что на пике что-то сгибается, думаю это вселенная.
>>169362
Ну если бы работало?
Аноним 17/04/15 Птн 12:23:23 #77 №169612 
>>169386
Развивающий вопрос:
У меня складывается впечатление что кротовая нора почти то же самое что и ЧД. Тогда возникает вопрос "Где сингулярность, блеать?!" и далее: ну допустим что то отожралось до таких размеров что смогло искривлять пространство@время но по каким критериям тогда выберется точка "выхода" ? Я конечно понимаю что это все теория но раз вскрыли эту тему...
Аноним 17/04/15 Птн 12:32:21 #78 №169614 
>>169612
>кротовая нора почти то же самое что и ЧД
Нет, они из разных теоретических доктрин, так сказать.
Аноним 17/04/15 Птн 13:59:53 #79 №169623 
>>169614
то есть в о вселенной не может существовать черных дыр и нор одновременно?
Аноним 17/04/15 Птн 14:58:06 #80 №169633 
>>169623
Могут, просто ЧД не равно КН, а КН не равно ЧД. ЧД по некоторым гипотезам может обладать свойствами КН. Вообще, рекомендовал бы ознакомится со статьей в вики по ЧД - там и интересно и исчерпывающе.
Аноним 17/04/15 Птн 15:05:19 #81 №169634 
>>169633

быстрофикс
Аноним 17/04/15 Птн 16:17:50 #82 №169652 
>>169386
Это что, шутка?
Аноним 17/04/15 Птн 16:20:43 #83 №169653 
Есть ли в галактике/вселенной места где локальная гравитация пренебрежительно мала? Кроме лагранжевых точек.
Аноним 17/04/15 Птн 16:33:01 #84 №169658 
>>169653
Что значит места? Лучше было бы сказать "системы отсчета". Пренебрежимо мала она на достаточно высокой орбите любого объекта, читай везде где приливные силы пренебрежимо малы. Потому что ускорение от гравитации принципиально ничем не отличается.
Аноним 19/04/15 Вск 00:16:00 #85 №170003 
Посоны, поясните, как работают боковые ускорители на Шаттлах? inb4: пщщщщ-ба-бах Там же внутри порошок, каким образом он поступает в джвигатель? И как не детонирует весь сразу?
Аноним 19/04/15 Вск 00:31:37 #86 №170006 
>>170003
На английском. C первой минуты наглядно:
https://www.youtube.com/watch?v=juJhOC_ByQk
Аноним 19/04/15 Вск 00:51:57 #87 №170012 
14293939173520.gif
14293939173521.png
>>170003
Пидорская шатаба ругается на спам-лист. Вот ответ пикчей.
Аноним 19/04/15 Вск 01:04:58 #88 №170020 
>>170012
Охуенно пояснил, спасибо.
другой мимокрокодил
Аноним 19/04/15 Вск 01:17:24 #89 №170023 
14293954441230.png
>>170006
>>170012
Спасибо.

А макака реально пизданулась.
Аноним 19/04/15 Вск 02:15:24 #90 №170039 
>>170023
Да, по сути SRB это большая бочка с порохом и дыркой. Только у неё очень сложная методика заправки (избежание полостей, комков, неравномерности и т.п.) и термодинамика сгорания.

Для охлаждения чаще всего используют аблятивную защиту на внутренностях канала, она испаряется и часть тепла уносит с собой, а также формирует подушку, защищающую внутренности от прямого контакта с горячим выхлопом. У неё тоже хитровыебанный профиль укладки.
Аноним 19/04/15 Вск 04:44:51 #91 №170065 
14294078915330.jpg
Путин когда на днях с народом общался, что-то про новую космическую станцию говорил, с которой 100% России видно будет.
Есть какие-нибудь подробности?
Аноним 19/04/15 Вск 05:17:54 #92 №170066 
>>170065
Наверное он про эту говорил, смотри ОП-пост.
https://2ch.hk/spc/res/142715.html
Аноним 19/04/15 Вск 05:36:06 #93 №170067 
>>170065
Фантазия чинуш, про которую только ленивый не сказал в последние месяцы, луркай. По смыслу приближается к ношению круглого, качению квадратного и чистке старта зубной щеткой. Невежество про "видно территорию" это что-то уровня экологически чистых ракет (точнее хуже, там хоть какое-то обоснование было, гидразин и АТ оче ядовиты).
> Есть какие-нибудь подробности?
Только твитерные разговоры да пара предложений от директоров КБ. ФКП-2025 уже подготовлена, ничего подобного там нет. Эту бы хоть наполовину исполнили, а не как прошлую.
Аноним 19/04/15 Вск 05:44:23 #94 №170068 
>>170065
Лол.
Аноним 19/04/15 Вск 10:28:41 #95 №170085 
Есть верхний и нижний пределы длинны световой волны? Дискретна ли энергия, принимаемая фотоном?
Аноним 19/04/15 Вск 21:07:51 #96 №170205 
14294668712480.png
http://geektimes.ru/post/249212/
Недавно группа каких-то укурков отсканила 100к галактик, и сделала вывод, что иных цивилизаций не найдено, лол.
Почитал про них, почитал комменты, и знаете - проиграл, и с укурков, и с комментаторов.

Укурки основывают свои фантазии на том, что развитым цивилизациям зачем-то нужна энергия солнца, лол.
Согласен с комментаторами - это уровень представлений "из пушки - на луну".
Не, энергия конечно нужна, но это же развитые цивилизации, наука, все дела. Даже сейчас, находясь в начале развития, мы нащупали бесконечное количество энергии в вакууме, и пусть нам она недоступна, но уж развитые цивилизации, полагаю, найдут как ее добыть, и получат аналог модуля нулевой точки из вселенной SG.
Далее, были доводы о том, что большие количества энергии не означают большого количества тепла. Комментаторы обосрали это доводы законами термодинамики, что по моему довольно глупо с их стороны: при чем тут термодинамика? Откуда берется тепло? При рассеивании энергии на сопротивлениях, не обязательно электрических. Вам знакомы сверхпроводники, имеющие нулевое сопротивление? Наверняка. А развитые цивилизации, чем они хуже нас? Неужто только мы можем додуматься до сверхпроводников?

Т.е. по факту укурки искали не иные цивилизации, а каких-то мутантов, имеющих уровень развития 18-го века, но зачем-то возводящие сферы дайсона. Это же явный омск: космические корабли с паровыми котлами, лол, бороздящие просторы космоса. И укурки об этом не догадались, лол. Каких уебков только не берут в науку, даже стыдно как-то, ей богу.
Ну а комментаторы с этой хабропараши оказались настолько тупыми, что начали закидывать разумные доводы религией (а фанатичная привязанность к науке религией и является) и антропоцентризмом. Это вообще занавес. Лично у меня после этого к хабропараше стойкое отвращение - аналогичное там наблюдается во всех тредах. Сборище догматиков какое-то.

Но самое главное, зачем я сюда и пишу - ни укурки от науки, ни догматики с хабропараши не подумали о третьем варианте, а именно: это же развитые цивилизации, с чего кто-то вообще взял, что они должны оперировать сотнями энергии?
Задумайтесь вот о чем: как бы выглядела цивилизация, существа которой имели бы развитую науку и технологии, имели бы доступ к любым количествам энергии локально, на халяву, и без тепловых потерь?
Походу большинству промыли мозги SW - почему-то все думают о Корусанте, планете-городе, где везде грязь, выделяется большое количество тепла.
Но прочитайте вопрос еще раз. С чего вы взяли, что этим существам понадобятся города, при таком-то уровне технологий? С такими технологиями любая избушка посреди тундры - комфортнее пятизвездочного отеля. Да им и дома могут оказаться не нужны - зачем?
Задумайтесь - зачем нам были нужны технологии? Чтобы защититься от природы, чтобы получить независимость от нее. Для этого мы и начали строить убежища, заселять пещеры, а позже - возводить свои личные пещеры на любой местности.
Но имея развитые технологии от непогоды и холода защититься не сложно - можно надеть браслетик, генерирующий какой-нибудь щит от дождя, ветра и снега, обогревающий тело. Или вообще перейти на уровень нанороботов, делающих тоже самое, но изнутри. Заметьте - я тоже оперирую уже известными нам концепциями, но, в отличии от догматиков, не исключаю и неизвестных: надо заранее принять, что мы многого не знаем.
Второй фактор технологий - транспорт и связь. При наличии технологий, науки и энергии, для этого опять же не нужна какая-либо инфраструктура: транспортировка может вестись направленными проколами пространства, телепортами (свертка пространства), или наносборкой - это только то, что нам известно. Для всего этого, при наличии энергии, не нужны ни дороги, ни транспортные аппараты, линии энергопередачи (энергия генерируется локально, в точке потребления), здания, и т.п. Связь же - даже мы уже имеем портативные личные коммуникаторы, так что развитые цивилизации со связью явно не будут испытывать проблем.
Третий фактор - производство. Но опять же - оно нужно только нам, чтобы преобразовывать окружающий мир, ради того, чтобы выжить. При наличии энергии и технологий в производстве необходимость пропадает - все что угодно могут собирать нанороботы. При наличии науки - не нужна даже сама материя, т.к. ее могут генерировать те же наносборщики прямо по месту печати. Т.е. у таких цивилизаций любой предмет буквально может быть получен из воздуха - для нас это кажется магией, даже сейчас, но это всего-лишь чуть более развитые технологии и наука. И при таком производстве все заводы могут находиться непосредственно в теле любого существа, т.е. не иметь каких-либо заметных проявлений во внешнем мире: захотел получить вкусняшку - протянул руку, подумал о ней, наносборщики вылезли из руки, собрали на ней вкусняшку, ты ее схавал, как ни в чем не бывало. Но это так, утрирую - энергоснабжение организма, ремонт, питание, дыхание, удаление отходов - все это берется под контроль наносборщиками еще на первом этапе, т.е. существо с таким уровнем технологий не будет испытывать необходимость есть, пить, спать, испражняться, дышать, умирать. Оно просто будет жить. Его нельзя будет убить, без его согласия: в случае повреждений, тут же будет произведен ремонт, в случае угрозы глобального уничтожения (например удар огромного астероида по голове) - мгновенная переброска тела в безопасную точку пространства, в случае более слабых воздействий - просто усиление свойств кожи или постройка над ней наноскелета, а то и полноценной брони.

Т.е. подобные существа смогут жить на природе, в естественном виде, как обычные животные, наслаждаться полнотой жизни, а не обслуживать технологии, как мы - это технологии их будут обслуживать.
Т.е. для любого, при взгляде извне, эти существа не будут отличаться от неразвитых животных - никаких городов, никаких источников энергии, никаких преобразований окружающей природы под себя, мегалообьектов, мощных источников излучения - вообще ничего того, что мы ожидаем найти, когда ищем технологически продвинутую цивилизацию, братьев по разуму. Зато при внимательном взгляде, мы обнаружим, что эти существа владеют какой-то магией - например не умирают, не едят. Хотя кто сказал, что они не будут есть ради удовольствия? Технически - они вообще могут не отличаться от неразвитых существ, могут не показывать свою магию, если не хотят: например при убийстве могут телепортироваться, но оставлять мертвую копию тела убийцам - с точки зрения убийцы существо умерло, с точки зрения существа - оно выжило, и обмануло убийцу.
Т.е. развитые технологии позволят существам вернуться в естественную среду обитания их предков (вплоть до того, чтобы воссоздать ее - переработать города обратно в леса), но, в отличии от предков, подарят потомкам магию, абсолютную власть над природой.
И самое смешное - мы спокойно пройдем мимо таких существ, даже не обнаружив следов их присутствия (помним - из-за технологий они могут путешествовать где хотят, и обитать даже на непригодных для жизни планетах, или вообще в космосе). Пройдем мимо тех, кого так долго искали. Потому что ищем не там.
Аноним 19/04/15 Вск 21:34:30 #97 №170208 
14294684710430.png
Почему спутники падают на Землю, а Луна не падает?
Аноним 19/04/15 Вск 22:04:54 #98 №170209 
>>170205
Годно. Схоронил.
Аноним 19/04/15 Вск 22:12:26 #99 №170211 
>>170208
Она тяжелая.
Аноним 19/04/15 Вск 22:30:16 #100 №170213 
>>170208
>спутники падают
летают в термосфере, летают быстро

>Луна не падает
летает в вакууме, летает медленно
Аноним 19/04/15 Вск 22:32:09 #101 №170214 
>>170211
Ее тяжесть спрессовывет вакуум, и стоит на нем, как на прочном фундаменте.
Аноним 19/04/15 Вск 22:40:12 #102 №170216 
14294724125470.gif
>>170208
Луна даже не то что не падает, а наоборот отдаляется. Правда OCHEMEDLENNO, по несколько см в год. Происходит это за счет приливного ускорения, тратя энергию вращения Земли. Когда-то давно она была намного ближе.

А спутники недостаточно тяжелые для приливного ускорения, их пертурбации сталкивают с орбиты.
Аноним 19/04/15 Вск 22:42:11 #103 №170218 
>>170205
Охуенная паста, которой тем не менее вместе с прочей кардашевщиной место в /sf/ скорее
Аноним 19/04/15 Вск 22:44:03 #104 №170219 
>>170216
Т.е. Луна разгоняется, тормозя Землю, лол. Она ворует нашу энергию, сучечка такая.
Аноним 19/04/15 Вск 22:49:20 #105 №170220 
Ананасы, объясните пожалуйста. Как видит глаз? какова роль фотонов? мы видим всегда в прошлом?
Аноним 19/04/15 Вск 22:53:33 #106 №170221 
>>170213
>летает в вакууме
Вакуума не существует. Любую пустоту равномерно заполняет газ.
В этом вакууме газ под низким давлением, по 1 молекуле на квадратный сантиметр. Но и поперечная площадь луны отнюдь не один квадратный метр - парусность у ней огромная, так что и силы, ее тормозящие из-за трения о газ, наверняка очень огромные - сотни и тысячи тонн.
И пусть, по сравнению с ее массой и энергией эти силы - капля в море, это нисколько не умаляет их огромной абсолютной величины.
Аноним 19/04/15 Вск 22:54:52 #107 №170222 
>>170220
Ну, представь, что тебе в драке зарядили кулаком в глаз - ты увидел звездочки. Вот примерно также и глаз видит, только вместо кулака - фотоны, а вместо звездочек - электроимпульсы.
Гугли фотоэффект.
Аноним 20/04/15 Пнд 01:27:09 #108 №170239 
14294824300090.jpg
>>170205
>Наука
>Технологии
>Развитая цивилизация
А не пойти бы тебе нахуй? Ещё и кудахчешь что-то про укурков и догматиков. Нахуй, быстро и решительно.
Аноним 20/04/15 Пнд 01:29:06 #109 №170240 
>>170211
не тяжёлая она нихуя.
Аноним 20/04/15 Пнд 01:31:42 #110 №170242 
>>170219
Вот почему на Земле пиздец как холодно, хотя раньше в Сибири бананы можно было выращивать. Сучья луна, ебал её в рот, презираю.
Аноним 20/04/15 Пнд 02:58:45 #111 №170246 
14294879251160.gif
>>170220

На пике схема активации зрительного каскада:
I – в темновом состоянии родопсин неактивен (R). a-Субъединица трансдуцина (Т) находится в комплексе с GDP (Ta-GDP) и связана с димером b- и g-субъединиц (Тbg). сGMP- фосфодиестераза (PDE) – гетеротетрамер, состоящий из двух гомологичных каталитических a- и b-субъединиц (PDEab) и двух идентичных g- субъединиц (PDEg), являющихся внутримолекулярными ингибиторами фермента, неактивна. Гуанилатциклаза поддерживает высокий уровень cGMP в цитоплазме. сGMP-зависимые катионные каналы в плазматической мембране находятся в открытом состоянии, и катионы Na+ и Са2+ могут диффундировать из внеклеточного пространства в цитозоль. Внутриклеточная концентрация Са2+ поддерживается на постоянном уровне находящимся в плазматической мембране Na+/Са2+, K+ -катионообменником;
II – в результате поглощения кванта света родопсин переходит в активное состояние (R -->R). Активный R связывается с трансдуцином и индуцирует обмен связанного с Тa GDP на GTP;
III – комплекс R-(Ta -GTP)-Tbg диссоциирует на R, T и активный комплекс Ta-GTP , после чего R способен активировать другую молекулу трансдуцина;
IV – Ta-GTP активирует PDE. Активированная фосфодиэстераза PDEab гидролизует множество молекул сGMP. Снижение внутриклеточной концентрации сGMP приводит к закрытию cGMP-зависимых каналов, что влечет за собой гиперполяризацию плазматической мембраны

Если нихуя не понял то:
Фотон попадает на сетчатку, которая содержит белок родопсин, и изменяет его структуру. Раздражение зрительного нерва происходит при фотолитическом разложении родопсина. Чувствительность человеческого глаза такова, что он способен реагировать на поток из 5-6 фотонов в секунду.
>мы видим всегда в прошлом?
Мы всегда видим прошлое, но скорость передачи и и обработки очень высока, так что отставание несущественно.
Аноним 20/04/15 Пнд 08:12:35 #112 №170253 
>>170246
а как формируется изображение в мозгу?
Аноним 20/04/15 Пнд 08:49:16 #113 №170254 
>>170253
В затылочной доли головного мозга, в зрительной коре. Процесс довольно сложный, тебе бы лучше к биологам за этим.
Аноним 20/04/15 Пнд 12:29:56 #114 №170270 
>>170254
я возможно не совсем правильно выражаюсь, понятно, что конечное изображение получается у нас в мозгу.
Фотоны, это частицы, вот они отразились от поверхности, попали в глазное яблоко, там они передали, я так понимаю, сигнал в нерв (ну грубо говоря). Т.е. они на себе несут само изображение, например (он конечно корявый), вот смотрим мы на поверхность, если разобьем её на мелкие квадраты (типа пиксели), вот фотон несет в себе информацию каждого пикселя?
Аноним 20/04/15 Пнд 12:55:43 #115 №170271 
>>170270
>вот фотон несет в себе информацию каждого пикселя?
Фотон не несет информацию пикселя. Фотон отразился от поверхности и полетел в глаз. Он имеет конкретную длину волны. Длинна волны, например, определяет цвет. При слабом освещении колбочки, которые вместо родопсина юзают йодопсин, не эффективны, поэтому действуют палочки. Они не могут в цвета, зато отлично реагируют на даже слабое освещение, тоесть на количество фотонов в единицу времени, и на основе темнее-светлее можно построить цельное изображение.
Аноним 20/04/15 Пнд 13:25:08 #116 №170274 
14295255081330.gif
>чувство, когда осознаёшь что вселенная бесконечна, но никто не может объяснить как
Аноним 20/04/15 Пнд 13:44:58 #117 №170277 
14295266986360.jpg
14295266986361.jpg
>>170274
Ты тот аутист из прошлого треда, который уже спрашивал про эту хуйню?
Аноним 20/04/15 Пнд 13:53:31 #118 №170280 
>>170277
Про это первый раз.
Аноним 20/04/15 Пнд 15:02:03 #119 №170284 
>>170271
благодарю анон
Аноним 20/04/15 Пнд 17:16:30 #120 №170297 
14295393904780.png
>>170221
>парусность у ней огромная
Парусность у твоей мамки огромная.

Я даже не поленился и картиночек принес.
"Парусность" пропорциональна квадрату радиуса, масса - кубу. Так что чем больше объект, тем слабее тормозит о среду.

Так что луне должно быть совсем похуй на жидкость твоего вакуума.
Аноним 20/04/15 Пнд 17:50:46 #121 №170298 
У меня вопрос по ядерным ракетным двигателям.
Удельный импульс двигателей зависит от скорости истечения рабочего в-ва. У двигателей на химическом топливе скорость истечения прямо зависит от температуры. Самые лучшие выдают температуру около 5000С емнип, а это скорость истечения 2-3 км/с и на этом развитие застряло.
Утверждают, что двигатели с ядреным топливом могут преодолеть этот барьер, но КАК они это сделают не расплавляясь к хуям? Ядерный реактор даже до 2000 не разогреешь, в чем тогда профит?
Аноним 20/04/15 Пнд 19:34:40 #122 №170310 
>>170298
>Утверждают, что двигатели с ядреным топливом могут преодолеть этот барьер, но КАК они это сделают не расплавляясь к хуям?

Например реакция может происходить в уже расплавленом, точнее испаренном реакторе - см газофазные реакаторы/двигатели.
Аноним 20/04/15 Пнд 19:38:18 #123 №170313 
>>170298
>Ядерный реактор даже до 2000 не разогреешь
>У двигателей на химическом топливе скорость истечения прямо зависит от температуры.

>Ядерный реактор
>на химическом топливе

По-моему ты просто немного запутался
Аноним 20/04/15 Пнд 19:40:53 #124 №170316 
>>170310
>реакция может происходить в уже расплавленом, точнее испаренном реакторе

Прошу прощения, этот реактор - получается, одноразовый что ли?
Аноним 20/04/15 Пнд 20:44:06 #125 №170326 
>>170205
Вот кстати все говорят о развитии и человечество постоянно растет и развивается. Но что будет когда оно разовьется до сверх цивилизации как описанная и уже некуда будет двигаться? Ну там происхождение вселенной известно, галактики захвачены, доказано существование эфира... Что дальше-то?
Аноним 20/04/15 Пнд 21:05:22 #126 №170337 
>>170326
А что сейчас?

Аноним 20/04/15 Пнд 21:13:14 #127 №170340 
>>170316
>Прошу прощения, этот реактор - получается, одноразовый что ли?

Ну он получается как камера сгорания ЖРД. А со стенками проблема, да, как и у термоядерного реактора.

https://en.wikipedia.org/wiki/Gas_core_reactor_rocket
Аноним 20/04/15 Пнд 22:13:19 #128 №170383 
>>170340
>термоядерного реактора
А такие уже существуют?
Аноним 20/04/15 Пнд 23:54:51 #129 №170441 
>>170383
Да. Локхид сделал уже.
Аноним 21/04/15 Втр 01:14:47 #130 №170454 
14295680873810.png
>>170270
А еще, охуей! Изображение получаемое сетчаткой - перевернутое, и дети впервые открыв глаза просто фшоке.

Но потом ничего, что-то у них в мозгу переворачивается и мир становится принципиально исследуемым
Аноним 21/04/15 Втр 01:15:41 #131 №170455 
>>170441
и даже Андрей Росси
Аноним 21/04/15 Втр 01:27:45 #132 №170460 
>>170454
А теперь ещё охуей. Если одеть специальные переворачивающие изображение очки, через несколько дней мозг адаптируется и видит нормально. Если очки снять придётся привыкать заново.
Отака хуйня, малята
Аноним 21/04/15 Втр 10:45:53 #133 №170493 
>>170460
Пиздёж и то и это.
Изображение на сетчатке было бы "перевёрнуто", если бы сама сетчатка не была.
Про охуительные эксперименты просто анекдот для воннабиучёных.
Аноним 21/04/15 Втр 10:50:55 #134 №170494 
>>170270
> фотон несет в себе информацию каждого пикселя?
Да, но не структурную информацию.
И одновременно нет, потому что луч отражается по волновой функции.
Аноним 21/04/15 Втр 11:01:20 #135 №170495 
>>170205
> как бы выглядела цивилизация, существа которой имели бы развитую науку и технологии, имели бы доступ к любым количествам энергии локально, на халяву, и без тепловых потерь?
Биологическая цивилизация, обитающая в нашем плане эфира просто захапала бы как можно большую территорию и летала от звезды к звезде по мере их угасания.

Другое дело, что источники энергии "в вакууме" просто не находятся в нашей эфирной луже. Мы видим круги на эфире, но их источник нам принципиально недоступен.

> пок-пок-пок магия
Ясно.

Ещё вотри про большой взрыв и чёрные дыры.
Аноним 21/04/15 Втр 11:26:00 #136 №170497 
>>170495
У нас было 2 пакета торсионных полей, 75 инерциоидов, 5 упаковок креационистских теорий, пол-солонки волнового генома Гаряева и целое множество гомеопатии всех сортов и расцветок, а также френология, телегония, ящик мичуринской биологии ,парапсихология , новая хронология Фоменко и эфир. Не то что бы это был необходимый запас для поездки. Но если начал собирать дурь, становится трудно остановиться. Единственное что вызывало у меня опасение - это эфир. Нет ничего более беспомощного, безответственного и испорченного, чем эфирные зомби. Я знал, что рано или поздно мы перейдем и на эту дрянь.
Аноним 21/04/15 Втр 12:55:54 #137 №170508 
>>170495
Кефироблядь приползла, из сайфача, шипя и извиваясь. А ну Брысь!!!
Аноним 21/04/15 Втр 14:34:16 #138 №170517 
Котаны, я тут бегло почитал доску, и понял что космос находится в незавидном положении, потому что он нахуй никому не нужен, потому что всё и так всех устраивает. С яблонями на Марсе мы уже соснули, остаётся только ждать, когда правительства забьют на финансирование МКС. Или я не прав? Какие вообще есть перспективы освоения космоса? Ну, кроме, разве что, топливного кризиса?
Аноним 21/04/15 Втр 15:08:08 #139 №170519 
14296180884090.jpg
>>170517
>топливного кризиса
Тут скорее наоборот, без горючки забьют не только на МКС с марсом, но и на навигаторы с тарелочным телевидением.

>нахуй никому не нужен
Совершенно верно, толчком к его освоению может стать только пренаселение Земли. Но так как популяция до сих пор саморегуируется низкой рождаемостью в богатых странах и высокой смертостью - в бедных, то перенаселение должно произойти с в разы более тяжелыми последствиями, чтобы в направлении космоса хоть что-то начинали принимать.
Да и то, прежде чем строить колонии на луне, надо хотя бы для начала заселить Сибирь с Антарктидой.
Между прочим, они гораздо более пригодны для жизни, чем тот же Марс.

>Какие вообще есть перспективы освоения космоса?
Орбитальные электростанции, например.
Заебатые херовины, сам посуди: и места не занимают, и работают почти круглосуточно.
https://ru.wikipedia.org/wiki/%D0%9A%D0%BE%D1%81%D0%BC%D0%B8%D1%87%D0%B5%D1%81%D0%BA%D0%B0%D1%8F_%D1%8D%D0%BD%D0%B5%D1%80%D0%B3%D0%B5%D1%82%D0%B8%D0%BA%D0%B0
Аноним 21/04/15 Втр 15:13:35 #140 №170520 
>>170517
>Котаны, я тут бегло почитал доску, и понял что космос находится в незавидном положении, потому что он нахуй никому не нужен, потому что всё и так всех устраивает. С яблонями на Марсе мы уже соснули, остаётся только ждать, когда правительства забьют на финансирование МКС. Или я не прав? Какие вообще есть перспективы освоения космоса

Ты неправильно дядя федор бутерброд ешь, то есть spc читаешь. Фраз "нетзадач" происходит не от того что космос ненужен. А от того что то, что есть в космосе, не нужно за такие деньги. А таких денег оно стоит потому что технологии человеков крайне неразвиты. И эту проблему просто вбухиванием бабла в космос не решить - у человеков не чего то отдельного нехватает, а общетехнический уровень слишком низкий. А эта проблема решается только развитием технологий вообще.

Отсюда следствия:

1) Пока ситуация вот такая, финансирование космоса(за исключением практических применений) идет по тому же принципу что и фундаментальной науки - как инструментария для прикладников. То есть как некий небольшой процент от прикладных исследований.

2) Врятли МКС закроют - это чтото вроде андронного коллайдера. Более того, по МКС наглядно видно готово ли человечество шагать дальше и насколько далеко.

3) когда технологии дойдут до нужного уровня мы это увидим: стоимость операций в космосе(на луне, орбите, итп) в расчете на возможности упадет до некоего предела, что приведет к тому что станет разумно вбухивать в космос средства как коммерческим так и некоммерческим методом. Это приведет к космическому пузырю, на подобие дотком-пузыря.

3.1) Есть мнение что с частной космонавтикой мы стоим уже на пороге этого пункта,

4) во всей этой дискусии начинают набегать порашники и вмурло , потому что по сути вопрос освоения космоса - это вопрос экономико-технологический. Рассуждения о технологиях без реального экономического контекста тут абсолютно бессмысленно - а в этом плане полно непоняток. Например та же стоимость пусков РН за редкими исключениями высасывается из пальца.
Аноним 21/04/15 Втр 15:48:55 #141 №170523 
>>170520
Как ты умудряешься делать по пять ошибок в одном предложении?
Аноним 21/04/15 Втр 15:49:14 #142 №170524 
14296205546430.jpg
Существуют ли принципиальные ограничения МКС по принципу пикрелейтед?

Алсо, возможна ли искусственная гравитация по типу одиссеи или интерстеллара, т.е. маленькое кольцо, сравнимое с кораблем создает притяжение в главном отсеке?
Аноним 21/04/15 Втр 15:52:04 #143 №170525 
>>170524
>Существуют ли принципиальные ограничения МКС по принципу пикрелейтед?
Бабло и сложность сборки.
>возможна ли искусственная гравитация по типу одиссеи или интерстеллара
Да, возможна.
Аноним 21/04/15 Втр 15:58:13 #144 №170526 
>>170523
>Как ты умудряешься делать по пять ошибок в одном предложении?

Потому что я убийца граманацистов.
Аноним 21/04/15 Втр 16:06:50 #145 №170533 
>>170520
>космос находится в незавидном положении, потому что он нахуй никому не нужен, потому что всё и так всех устраивает.
Освоение космоса неизбежно. Возможно очень и очень не скоро, но это неизбежно. Человечество на пути своего развития обязательно переселится на искусственно созданные планеты, кольца и т.п. Таков сам принцип человеческого мышления, его суть.
В какой-то момент люди перестанут полагаться на непредсказуемо летающие в межзвездном прастранстве шары материи, которые сами представляют огромную опасность своими ебанутыми геологическими процессами типа внезапной смены полюсов, взрывов супервулканов и т.п., не говоря уже о падении астероидов.
Люди стремятся колонизировать Марс или Луну, не потому что на Земле нет места, а потому что хотят создать собственную планету.
Аноним 21/04/15 Втр 16:37:00 #146 №170543 
14296234202640.png
>>170383
А как же.
Аноним 22/04/15 Срд 00:34:14 #147 №170621 
Посонтре, а ведь в солнечной системе орбиты планет лежат в одной плоскости. Глянул про млечный путь - та же херь, плюс-минус.
А почему так?
Аноним 22/04/15 Срд 00:36:32 #148 №170623 
>>170524
Тут чисто технически мне не ясно как реализовать переход из зоны невесомости в зону, которая куртитца. Только всю эту бодягу останавливать. А потом запускать. На 8 раз она стопудняк не запустится и все будут по-старинке рассекать по коридорам.
Аноним 22/04/15 Срд 02:00:55 #149 №170627 
>>170623
>как реализовать переход из зоны невесомости в зону, которая куртитца.
Каком, зачем переход городить. Ебала крутится целиком.
Аноним 22/04/15 Срд 07:30:22 #150 №170639 
>>170621
суммарный момент импульса не равен нулю.

если был бы равен, то все звезды/планеты рано или поздно столкнулись бы в общем центре масс
Аноним 22/04/15 Срд 08:36:32 #151 №170641 
Чем точки лангража примечательнее, чем просто круговые геостационарные орбиты?
Аноним 22/04/15 Срд 08:47:40 #152 №170642 
Кстати, а что из себя представляет шаровое скопление, оно ведь шаровое, следовательно не должно особо вращаться чтоб не перейти в форму диска, но в то же время звезды не слетаются в центре, это что, некое относительное гравитационное равновесие на относительно коротком промежутке времени (равным истории наших наблюдений)?
Аноним 22/04/15 Срд 09:56:07 #153 №170648 
Как повлияло бы на Землю полное отсутствие луны и каких либо спутников?
Аноним 22/04/15 Срд 09:59:38 #154 №170650 
>>170648
Хреново. Жизни в нынешнем ее виде не было бы.
Аноним 22/04/15 Срд 12:12:11 #155 №170657 
>>170627
а челнок типа синхронизировать с вращающейся хуйнёй? Ну можно.
Аноним 22/04/15 Срд 12:40:57 #156 №170660 
>>170621
Потому что солнечная система образовалась из крутящегося говна, а крутящееся говно всегда схлопывается в диск.
Аноним 22/04/15 Срд 15:01:57 #157 №170675 
>>170648
Приливы в два раза реже и в два раза меньше.
Аноним 22/04/15 Срд 15:07:57 #158 №170676 
>>170642
В позапрошлом треде спрашивали про шаровые скопления.
>>163328
Звезды там движутся от периферии к центру и обратно по вытянутым сложным орбитам. Почему шаровые скопления именно шаровые точно не известно. Известно, что звезды в этих скоплениях очень старые, вероятно такие образования довольно устойчивы в течении очень долгого промежутка времени.
Аноним 22/04/15 Срд 18:44:15 #159 №170688 
14297174550310.jpg
Предположим, что завтра к Марсу (или просто в дальний космос) уходит с орбиты межпланетный корабль. Гравитационное поле Земли, насколько мне известно, способно влиять на тела, находящиеся примерно в 1 млн км от планеты, т.е. около трёх радиусов Луны.
Вопрос. Означает ли это, что недели через три-четыре после отлёта корабль пересечёт своеобразную точку невозвращения?
Пока корабль находится в сфере гравитационного влияния Земли, его можно будет в случае необходимости развернуть кормой вперёд и погасить скорость с таким расчётом, чтобы разомкнутая парабола снова схлопнулась в эллипс, и вернуться обратно на Землю. Но после выхода в межпланетное пространство прямое возвращение станет невозможным? (По крайней мере, не раньше, чем корабль один раз обернётся вокруг Солнца по солнечной орбите). Я правильно мыслю, или I've played too much KSP?
Аноним 22/04/15 Срд 18:54:13 #160 №170689 
>>170688
> пик
Марсианский проект Королёва полвека спустя?
Аноним 22/04/15 Срд 18:54:52 #161 №170690 
>>170688
> находящиеся примерно в 1 млн км от планеты
Да больше
Аноним 22/04/15 Срд 19:16:35 #162 №170694 
>>170689
Я сам удивился, когда нашёл (только что). У меня KSP от этого. Там ничего не переломится между блоками, когда они разгоняться от Земли будут? А вот лендер понравился, годный вариант.
Аноним 22/04/15 Срд 22:08:19 #163 №170723 
>>170688
> Означает ли это, что недели через три-четыре после отлёта корабль пересечёт своеобразную точку невозвращения?
Нет, развернуть траекторию ты можешь в любой момент, если дельты хватит. Сама по себе сфера Хилла ничего не значит, например ты можешь статически находиться в её пределах, но динамически - за пределами гравитационного влияния Земли (иметь скорость выше второй космической). Если дельты хватит, ты можешь хоть как в НФ летать - напрямую, без всяких гомановских переходов и эллиптических орбит, полдороги разгоняться, полдороги тормозить, лол.
Аноним 22/04/15 Срд 22:21:25 #164 №170728 
14297304857420.png
>>170657
Прикладываю пик. Сам нарисовал.

Крутятся два кольца одно в одну сторону другое соответственно в другую отсек стержень висит на месте все счастливы.

Ебля со стыковкой и уравниванием скоростей стыкуемого говна нинужна.

Где моя нобелевка сученьки?
Аноним 22/04/15 Срд 22:27:59 #165 №170731 
>>170728
Где твой пик, лауреат хуев?
Аноним 22/04/15 Срд 22:32:02 #166 №170733 
>>170731
у макаки спроси у меня все отобразилось

Или ты в очи ебешься
Аноним 22/04/15 Срд 22:33:15 #167 №170735 
>>170733
Макака, почему у меня картинка сразу не отобразилась?
Аноним 22/04/15 Срд 22:45:42 #168 №170741 
>>170728
Нахуй не надо два кольца - момент сил стремится повернуть всю эту гирю, уравновесить их сложно.
Лучше использовать принцип гиродинов: одно обитаемое кольцо, и по обоим сторонам - два тяжелых, но маленьких, гиростабилизоторов, которые во первых поддерживают равномерность вращения обитаемого кольца, во вторых уравновешивают его, если масса начнет меняться, компенсируют все всплески массы, ну и со временем неравномерность может вырасти до предела их инерциальной емкости, и их придется разряжать, как и обычные гиродины.
Аноним 22/04/15 Срд 23:05:52 #169 №170755 
14297331525270.png
14297331525301.png
>>170741
И как же их разряжать, няша? Всех обитателей нахуй с поезда? мне гиродины разряжать надо?

и где блядь картинки? ты не на базаре а на имаджборде!

Первый пик - мое понимание того что сказал ананас выше по поводу момента сил

второй - то что этот ленивый хуйлан предложил

Аноним 22/04/15 Срд 23:08:39 #170 №170757 
>>170755
>>170728
Вы ебанашки. Нахуй не нужны подвижные части. Это усложнение и удорожании конструкции. Хуйня должна вращаться целиком и только так.
Аноним 22/04/15 Срд 23:14:42 #171 №170759 
>>170757
Чиновнику Роскасмаса припеклоу

А вот представь теперь что тебе надо на эту станцию что то доставить. Как к ней стыковаться если она крутится как шлюха?

А если место действия орбита Нептуна, к примеру? Мне, сука, что каждый раз надо тратить топливо на раскрутку - стыковку - торможение раскрутки каждый раз когда надо попасть на станцию чтоб чихнуть? топлива на эту хуйню ты сам туда доставишь?

Аноним 22/04/15 Срд 23:27:44 #172 №170762 
>>170759
>А если место действия орбита Нептуна, к примеру?
Смотрите сайфачер уже доставил туда невъебенную крутящуюся ебалу и у него проблемы теперь туда доставить 2 тонны хавки, топлива у него мало на синхронизацию челнока с ебалой видите ли. У тебя дырка в черепе, через нее логика подтекает.
Аноним 22/04/15 Срд 23:40:01 #173 №170765 
>>170676
Спасибо большое!
Аноним 23/04/15 Чтв 00:11:47 #174 №170774 
>>170759
>Как к ней стыковаться если она крутится как шлюха?
Как будто проблема. В орбитере например это типовая задача - стыковка к лунной станции, которая вращается целиком.
На хорошо отбалансированном корабле это не проблема - выравниваешь оси, выравниваешь угловые скорости, и станция кажется неподвижной, легкий пук по продольной оси - стыковка закончена.
Аноним 23/04/15 Чтв 00:31:26 #175 №170782 
>>170759
Миклуха смог и мы сможем.
Аноним 23/04/15 Чтв 00:32:14 #176 №170784 
>>170762
смотрите шкальник ничего еще никуда не доставил, а уже ищет дырки в черепах через которые можно соснуть немного логики

Челноку то норм крутится как шлюхе? любому челноку? или там астронавтов к потолку прихуярит центробежкой?
Аноним 23/04/15 Чтв 00:47:45 #177 №170789 
>>170774
да это все понятно, можно и под мостами на самолетах летать и хуй на лбу выращивать если очень надо, непонятной только нахуй эта ебля со стыковочным пилотажем нужна.

Трата топлива на раскрутку/торможение вращения, там где его может и взять то негде это только одна из очевидных видимых проблем.

-А космический аппарат который ты стыкуешь будет расчитан на то чтоб его вращать вокруг оси без риска для экипажа и оборудования?

- А если маневровые сопла/двигатели на челноке выйдут из строя (даже если какой то один наебнется ты вместо вращающегося челнока получишь эсцентрик и не пристыкуешься)

- А если стыковочный шлюз челнока при вращении вообще не в центре находится (ну конструкция неподошла хули, челнок старый попался)

- А если сраному космонавту приспичит наружу вылезти поссать устранить неисправность шлюза на этой крутящейся ебале, и его закономерно упиздячит в космические перди центробежкой (ТБ не усвоил, пристегнуться тросом забыл хули ты хотел) Ты его мамке что будешь говорить? что выебешь небось

- И как страховаться вообще в этом случае? тросс куда цеплять? на крутящуюся ебалу? Да вы извращенцы ребята!

Да дохуя всего. Нет блядь, будем экономить на двух сраных подшипниковых соединениях и жрать говно.
Аноним 23/04/15 Чтв 00:54:10 #178 №170797 
>>170784
Очень смешно, когда какой-то имбецил из /б/начинает детектить школьников. В 1985 году Джанибеков стыковал Союз с неуправляемой, вращающейся Салют-7 в ручном режиме.
Аноним 23/04/15 Чтв 00:58:56 #179 №170801 
>>170789
>Трата топлива на раскрутку/торможение вращения
Копейки
>А космический аппарат который ты стыкуешь будет расчитан на то чтоб его вращать вокруг оси без риска для экипажа и оборудования?
Если есть RCS - значит рассчитан
>А если маневровые сопла/двигатели на челноке выйдут из строя
Они трижды продублированы + есть гиростабилизаторы, как в самолетах - там они занимаются именно этим, тонкой юстировкой.
>А если стыковочный шлюз челнока при вращении вообще не в центре находится
Можно перенести ЦТ, либо сместить ось вращения динамически - компьютер потянет такие расчеты в реальном времени, и сможет удержать новую ось.
>А если сраному космонавту приспичит наружу вылезти поссать устранить неисправность шлюза на этой крутящейся ебале, и его закономерно упиздячит в космические перди центробежкой
А если ты из окна выпадешь и ногу сломаешь? Теперь все окна позаколачивать?
>И как страховаться вообще в этом случае? на крутящуюся ебалу?
А что тебя удивляет. Тебя послушать - там циркулярка о 30000 оборотов в минуту.
>Да дохуя всего. Нет блядь, будем экономить на двух сраных подшипниковых соединениях и жрать говно.
Если надежнее целиковая конструкция - будем экономить, что ж поделать. В любом случае для прибывающих кораблей похуй, крутится станция или нет - на то они и космические корабли, чтобы управляться прецезионно, им это медленное вращение - как нехуй делать, они и не такие фокусы могут вытворять, например могут пристыковаться к той же крутящейся ебале перпендикулярно оси вращения, как нехуй делать.
Аноним 23/04/15 Чтв 01:02:11 #180 №170805 
>>170797
эта история конечно заебок, мой милый дурачек из spc, но то была внештатная ситуация. я тебе говорю о случае когда в штатном порядке, когда космонавту надо производить стыковку чуть ли не чаще чем таксисту парковаться.

Ты разницу чувствуешь вообще?
Аноним 23/04/15 Чтв 01:03:08 #181 №170806 
>>170805
Съеби уже, тебе ясно сказали, что с этим проблемы только у тебя в голове.
Аноним 23/04/15 Чтв 01:04:48 #182 №170807 
>>170805
В штатном режиме на себя все возьмет автоматика, и вопрос перетекает в такую плоскость: может ли автоматика это или нет.
Если может - значит и голову нечего морочить.
Если нет - надо думать, что проще сделать: тормозить докпорт или научить автоматику.
Аноним 23/04/15 Чтв 01:08:02 #183 №170811 
>>170807
>может ли автоматика это или нет.
Может ли автоматика сравнять угловые скорости с объектом на известном расстоянии? Даже диды это освоили 30 лет назад.
Аноним 23/04/15 Чтв 01:11:08 #184 №170814 
>>170811
Если все так просто, хуле тогда мериканцы до сих пор вручную летают и стыкуются? Хуле шаттл не мог в автоматику?
Аноним 23/04/15 Чтв 01:17:58 #185 №170816 
14297410785610.jpg
>>170814
>хуле тогда мериканцы до сих пор вручную летают и стыкуются? Хуле шаттл не мог в автоматику?
Аноним 23/04/15 Чтв 01:19:02 #186 №170817 
>>170814
Не знаю, что там за проблемы у американцев, в СССР с начала 70-х стыковались на "Игле", а с 1986 стыкуются "Курсом".
Аноним 23/04/15 Чтв 01:24:35 #187 №170820 
>>170801
>Копейки
Орбита Земли копейки, орбита Марса уже полновесные рубли, на орбите Нептуна уже половина твоей зарплаты на полпшика соплом уйдет
>Если есть RCS - значит рассчитан

допустим


>Они трижды продублированы + есть гиростабилизаторы, как в самолетах - там они занимаются именно этим, тонкой юстировкой.

Заебок. Скажи мне, что этот тройной дубляж тоже копейки, я тогда спать крепче буду.

>Можно перенести ЦТ, либо сместить ось вращения динамически - компьютер потянет такие расчеты в реальном времени, и сможет удержать новую ось.

возможно. Надо нарисовать чтоб представить но мне лень сейчас.


>А если ты из окна выпадешь и ногу сломаешь? Теперь все окна позаколачивать?

Если я выпаду и сломаю, меня подберут и будут лечить, а космонавт в сраный космос упиздячит. Ох не зли меня.

>А что тебя удивляет. Тебя послушать - там циркулярка о 30000 оборотов в минуту.
Нигде я не говорил про количество оборотов. Впрочем оборотов там хватает чтобы развить центробежку по силе равную земному притяжению, а это значит что космонавту надо будет лезть к станции по страховочному тросу в скафандре преодолевая 1 же.

>Если надежнее целиковая конструкция - будем экономить, что ж поделать. В любом случае для прибывающих кораблей похуй, крутится станция или нет - на то они и космические корабли, чтобы управляться прецезионно, им это медленное вращение - как нехуй делать, они и не такие фокусы могут

Ну если я тебя чем обидел ты извини, но надежности я тут не увидел никакой, как и дешевизны.

Предлагается просто этот твой RCS в части вращения вокруг оси заменить на два сраных подшипника. Да там свои проблемы, трение всякое хуе мое, конструкция похитровыебаней будет. Зато выхлоп очевидный, эту хуйню один раз крутнул и больше не возвращаешься к этому вопросу месяцами и годами.
Аноним 23/04/15 Чтв 01:34:06 #188 №170822 
Возможна ли реакция между метаном и водородом?
Аноним 23/04/15 Чтв 01:34:56 #189 №170823 
>>170820
>Орбита Земли копейки, орбита Марса уже полновесные рубли, на орбите Нептун
Если ты притащил на Марс, или на Нептун крутящуюся хуевину, то долить два ведра горючки в челнок это тьфу и растереть, а не затраты.
>Скажи мне, что этот тройной дубляж тоже копейки
Тройной дубляж делают в любом случае для всех систем, синхронизировать вращение можно на стандартных двигателях стандартного корабля, а пилить какой-то новый для вращения.
>лезть к станции по страховочному тросу в скафандре преодолевая 1 же
>а космонавт в сраный космос упиздячит
Твой астронавт так же упиздячит, когда будет чинить твое колесо, надетое на невращающуюся ось. Так что починки при авариях так и так обернутся проблемами.
>но надежности я тут не увидел никакой, как и дешевизны.
Любая движущаяся деталь, или усложненный узел - это минус надежность и плюс цена. Основа инженерии блядь. У тебя явное усложнение конструкции, потому что у тебя есть эта деталь, а на целиковой станции ее нету.
>эту хуйню один раз крутнул и больше не возвращаешься к этому вопросу месяцами и годами.
Целиковую хуйню крути и не возвращайся к ней годами. Челнок пусть к ней прицеливается, а не она к челноку.



Аноним 23/04/15 Чтв 01:38:00 #190 №170825 
Как такая маленькая планета как Титан смогла удержать такую плотную атмосферу?
Аноним 23/04/15 Чтв 01:38:56 #191 №170826 
>>170822
>Возможна ли реакция между метаном и водородом?
Нет конечно.
Химию еще не проходили штоле?
Аноним 23/04/15 Чтв 01:44:35 #192 №170827 
>>170820
>Орбита Земли копейки, орбита Марса уже полновесные рубли, на орбите Нептуна уже половина твоей зарплаты на полпшика соплом уйдет
Блядь, скажи мне, что ты туда прилетел вообще без запаса на непредвиденные ситуации - засмею.
>Заебок. Скажи мне, что этот тройной дубляж тоже копейки, я тогда спать крепче буду.
Скажу, что он для тебя вообще бесплатен, и идет искаропки: иначе бы не было так удобно вертеться по трем осям, без трех групп одинаковых систем RCS.
>Ох не зли меня.
Проиграл.
>Впрочем оборотов там хватает чтобы развить центробежку по силе равную земному притяжению, а это значит что космонавту надо будет лезть к станции по страховочному тросу в скафандре преодолевая 1 же.
Ничего, пупок не развяжется, слазит. 1g - это не 100g, трагедию тут делать не из чего
>но надежности я тут не увидел никакой, как и дешевизны.
Попробуй загерметизируй вращающиеся соединения, да потом обслужи их в вакууме, без утечек драгоценной атмосферы.
А потом сравни это с неподвижными соединениями, которые сами по себе герметичны.
Для примера - глянь на конструкцию скафандров, суставы. Такая то высокотехнологичная ебала, и то часто сбоит.
>Предлагается просто этот твой RCS в части вращения вокруг оси заменить на два сраных подшипника.
Нахуя пилить какие-то лишние сущности, если системами RCS и так по дефолту укомплектованы все космические корабли - иначе они тупо не смогли бы принять нужную ориентацию, и хуй бы куда вообще улетели.
Т.е. ты выигрываешь пару м/с дельты, зато получаешь пару ненужных сложных узлов (на доставку которых также нужна дельта), которые могут пропускать атмосферу, которые сложно, но необходимо обслуживать. Короче получаешь лишний и большой гиммор, точку уязвимости, в которой рано или поздно случится протечка, которая может стоить гораздо дороже сэкономленных на уравнивании угловых скоростей копеек.
>Зато выхлоп очевидный, эту хуйню один раз крутнул и больше не возвращаешься к этому вопросу месяцами и годами.
Прикинь, без подшипников та же хрень - один раз крутнул, и все.
Аноним 23/04/15 Чтв 01:49:31 #193 №170828 
>>170823

Почему блядь прицеливаться то куда то надо обязательно.

устал я от тебя, няша

Завтра запилю конструкцию подешевле Заебал ты. Лишь бы денег на проект порезать
Аноним 23/04/15 Чтв 01:53:12 #194 №170829 
>>170825
>Как такая маленькая планета как Титан смогла удержать такую плотную атмосферу?
Во-первых атмосфера титана очень холодная и от того обладает большой плотностью. У поверхности она плотнее в 4 раза, чем у поверхности Земли. Во-вторых Титан довольно далеко от Солнца и так сильно уже не дует. В третьих, и в самых главных, атмосфера Титана постоянно обновляется за счет внутренних запасов под поверхностью, а так Титан теряет свою газовую оболочку, только очень медленно.
Аноним 23/04/15 Чтв 02:00:56 #195 №170831 
>>170829
Там есть метан и водяной лед, можно ли добывать топливо прямо на месте?
Аноним 23/04/15 Чтв 02:08:47 #196 №170838 
14297441279960.png
>>170828
>>170827
>>170823
вот блё
Аноним 23/04/15 Чтв 02:11:47 #197 №170840 
>>170838
Отлично.
Остался один вопрос. Как герметично соеденить вращающийся диск с неподвижным цилиндром, с запасом вращения на пару лет?
Аноним 23/04/15 Чтв 02:16:43 #198 №170843 
>>170840
думаешь надо прям герметично дохуя?

Может да. А зачем?

Воздух откачай с него делов то и не пользуйся шлюзовым отсеком если у тебя не стыковка, не?
Аноним 23/04/15 Чтв 02:17:43 #199 №170844 
>>170840
Шлифануть нулевкой плюс гелевая герметизация
Аноним 23/04/15 Чтв 02:20:07 #200 №170845 
>>170840
да и про запас вращения загнул ты дружище.

На кой ляд тебе пара лет.

Ты его крутить будешь три четыре раза в день по пол часа только в моменты стыковки.
Аноним 23/04/15 Чтв 02:21:30 #201 №170847 
14297448900720.jpg
>>170840
кароч выделяй бабло на проект и не беси своими претензиями, бюрократию тут развел, няшка
Аноним 23/04/15 Чтв 02:29:37 #202 №170849 
>>170843
>Воздух откачай с него делов то и не пользуйся шлюзовым отсеком если у тебя не стыковка, не?
Нет. Нужен отсек с невесомостью для научный исследований.
>Шлифануть нулевкой плюс гелевая герметизация
Так и представляю, как усатый Семен сидит с точилом и краской. Представляю, как это соединение разлетится на давлени 1МПа.
Аноним 23/04/15 Чтв 02:37:30 #203 №170853 
1. Правда ли что если рептилоиды спиздят солнце, сраная земляшка будет как ни в чем ни бывало вращается по орбите ещё восемь минут а плутонобляди целых пять часов?
2. Правда ли что обитаемая зона по мере старения солнца смещается к внешним границам системы и скоро надо заводить трактор?
3. Если утверждение N2 верно, можно допустить что например миллиард лет назад на Венере существовала цивилизация подобная земной, с двачами и битардами? Возможно ли теоретически отыскать материальные свидетельства существования венерических существ или все обратилось в тлен?
Я кончил
Аноним 23/04/15 Чтв 02:39:57 #204 №170854 
14297459974090.jpg
>>170849
Да ты че ты че
Аноним 23/04/15 Чтв 02:40:14 #205 №170855 
>>170849

маняоправдания пошли какие-то

в целиковой ебале есть отсек с невесомостью для исследований? конечно есть, в центре барабана невесомость

А усатый семен тебе чем неугодил?

Да и вообще развел тут, самая что ли сложная конструкция в спейсшипе?

Вон эти союзы наши летают с тремя ступенями. Дохуя сложно с точки зрения инженерии по мойму и ничего, и ничо семены справляются в целом.
Аноним 23/04/15 Чтв 02:46:16 #206 №170859 
>>170838
Нахуй тебе вообще стационарный шлюз? Ты не понимаешь, что в космосе нет ничего стационарного, и чтобы добраться до станции, кораблик и так должен покрутиться по всем осям 100500 раз, так что ему похуй, на пару оборотов больше или меньше, похуй, стационарна ли цель, или вращается.
Аноним 23/04/15 Чтв 02:48:25 #207 №170860 
>>170853
хуйню понаписал
первое - да, а потом я утонул в шуточках
Аноним 23/04/15 Чтв 02:49:35 #208 №170862 
>>170855
>конечно есть, в центре барабана невесомость
Её там нет. Та же центростремительная сила, разве что крайне мала. Однако, в биомолекулярных исследованиях это аукнется.
Аноним 23/04/15 Чтв 02:52:03 #209 №170864 
>>170853
1,2 - истино так.
3 - все может быть. Отыскать что-то можно лишь на истино мертвой планете, на которой все было заморожено на миллиарды лет неизменным. Но таких планет нет - даже луна подвергается многочисленным изменениям, ну а на планете с атмосферой отыскать следы цивилизации нельзя будет почти сразу после ее гибели. Даже на земляшке всего через несколько сотен лет после нас нихуя не останется - ни городов, ни машин, все обратится в пыль, лишь мегапамятники, типа пирамид, просуществуют чуть дольше, но все-равно вскоре будут поглощены эрозией и песками.
Что из пыли восстало, в пыль и уйдет - таково слово господне.
Аноним 23/04/15 Чтв 02:52:08 #210 №170866 
>>170859
хочу стационарный шлюз, заебал

одно дело покрутиться немного чтоб просто к шлюзу прислониться а другое дело еще вокруг своей оси раскручиваться.

Может челноку эта центробежка в хуй не впилась и вообще
Аноним 23/04/15 Чтв 02:56:03 #211 №170869 
14297469639130.jpg
>>170864
А допустим если они вышли в космос, какой-нибудь говноспутник на орбите Венеры (может даже с пикрелейтед внутри) сохраниться?
Аноним 23/04/15 Чтв 02:58:50 #212 №170870 
>>170866
Хуй с тобой. Навешиваешь на станцию над шлюзом краба на рельсах - пущай изображает из себя стационарного, герметизировать нахуй не надо.
Этот краб будет ловить корабль за докпорт, выравнивать угловые скорости, ориентировать его по шлюзу, и додавливать в шлюз, завершая стыковку.
Таким образом получится и рыбку сьесть и на хуй сесть: шлюз целиковый со станцией, не затронут, краб - отдельная хуита, без шлюза, торчит в космос, герметизации не требует, прикидывается для корабля стационарным шлюзом.
Таким образом краб не мешает автоматической стыковке, и сильно упрощает ручную, выравнивая скорости за пилота - пилоту надо только ткнуться в зону ответственности краба, и тот возьмет его за яйки, и закрутит как надо.
Аноним 23/04/15 Чтв 03:03:46 #213 №170873 
>>170870
А ещё можно гарпунами хуярить в корабли и потом тянуть за веревочку.
Аноним 23/04/15 Чтв 03:03:46 #214 №170874 
>>170831
>Там есть метан и водяной лед, можно ли добывать топливо прямо на месте?
Да, при чем прямо в жидком виде. Даже тратить энергию на сжиживание не надо. Прямо черпай из озера и разливай в тару. Но зачем?
Аноним 23/04/15 Чтв 03:04:48 #215 №170875 
>>170874
Чтобы назад вернутся. А лед все же надо сначала плавить а потом выделять кислород.
Аноним 23/04/15 Чтв 03:05:26 #216 №170876 
>>170869
Нет. Гравиколодцы не идеально ровные, плюс существуют всякие левые эффекты - спутник либо притянет к поверхности, либо выбросит с орбиты в ебеня.
Это как когда неровно крутишь в стакане жидкость - обязательно немного перельется через край, если спецом не балансировать. Но в космосе балансировать некому - вот и переливается.
Сохранится лишь жизнь - что-то, что способно будет все эти миллиарды лет выжить, и корректировать все это время орбиты. Автоматы, несмотря на свою прочность, не могут в авторемонт так, как живые организмы, так что в гонке со временем выиграет жизнь, а не автоматика.
Если сформулировать по другому: автоматы не могут гасить энтропию, тогда как жизнь только и питается энтропией.
Аноним 23/04/15 Чтв 03:07:59 #217 №170878 
>>170873
Проиграл в голосину.
Аноним 23/04/15 Чтв 03:09:08 #218 №170880 
>>170873
Можно
http://www.youtube.com/watch?v=nng6SB8uzlo
Аноним 23/04/15 Чтв 03:10:17 #219 №170882 
14297478173250.jpg
>>170873
Двачую
Аноним 23/04/15 Чтв 03:13:30 #220 №170883 
Если бы быдлокодеры писавшие проги для кассини-гюйгенс проебали запятую и аппарат шлепнулся с озеро с бензином, смог бы он что-то передать или булькнул не попрощавшись?
Аноним 23/04/15 Чтв 03:22:11 #221 №170884 
14297485311440.jpg
>>170883
>Зонд сел на твёрдую поверхность, хотя посадка в океане была также предусмотрена его конструкцией.
>Кассини» принимал сигналы «Гюйгенса» на этапе спуска в течение 147 минут 13 секунд и с поверхности — ещё 72 минуты 13 секунд до момента, когда орбитальный аппарат скрылся за горизонт. После этого сигналы зонда некоторое время принимались на радиотелескопе в Австралии, хотя и оказались слишком слабыми, чтобы использовать их в качестве канала передачи информации.
>Всего было передано более 500 мегабайт информации, в том числе порядка 350 изображений. Всего планировалось передать на Землю 700 фотографий, но из-за сбоя в компьютерной программе (предположительно, по причине ошибок при её разработке) половина изображений, переданных «Гюйгенсом», была утеряна.
>Изучение свойств грунта было осуществлено с помощью пенетрометра. Первоначально грунт интерпретировали как тонкую корку сравнительно однородной консистенции на более мягкой основе («крем-брюле»). Позже данные пенетрометра были пересмотрены: теперь считается, что при посадке он ударился о гальку, после чего погрузился в грунт, общая консистенция которого соответствует консистенции влажного песка или плотного снега. Зонд погрузился в грунт на глубину 10—15 см. При этом из грунта выделялся метан (его выбросы были зарегистрированы приборами зонда).
Аноним 23/04/15 Чтв 03:40:07 #222 №170888 
14297496078880.jpg
14297496078891.jpg
>Гигантский гексагон — не имеющий на сегодняшний день строгого научного объяснения атмосферный феномен на планете Сатурн. Представляет собой геометрически правильный шестиугольник с поперечником в 25 тыс. километров, находящийся на северном полюсе Сатурна.
Власти скрывают.
Аноним 23/04/15 Чтв 04:08:10 #223 №170889 
>>170888

Скорее всего это замерзший газ превратившийся в некое подобие северного материка дрейфующий в жидких слоях планеты!
Аноним 23/04/15 Чтв 04:08:41 #224 №170890 
>>170888
>Раструбили на всех сайтах
>Показали по дебилятору
>Власти скрывают.
Что спросить-то хотел?

> не имеющий на сегодняшний день строгого научного объяснения
Ох уж эти журналюги.
Аноним 23/04/15 Чтв 04:13:28 #225 №170892 
>>170888
http://old.computerra.ru/magazine/357574/
http://www.smekalka.pp.ru/forum/index.php?topic=5745.0
Аноним 23/04/15 Чтв 04:16:26 #226 №170894 
14297517869050.jpg
>>170888
Скrиваем, а ви таки шото имеете пrотив?
Аноним 23/04/15 Чтв 04:18:39 #227 №170895 
14297519190500.jpg
>>170888
Ви таки не повеrите, но мы и в земных условиях скrиваем.
Аноним 23/04/15 Чтв 04:21:40 #228 №170896 
>>170895
>>170894
ШОК!!! КОСМИЧЕСКАЯ БАЗА ЖИДОРЕПТИЛОИДОВ обнаружена на полюсе сатурна. Скачать без регистрации и смс
Аноним 23/04/15 Чтв 07:37:48 #229 №170908 
14297638680840.jpg
>>170883
Не ожидалось, что Гюйгенс вообще выживет, посадка была бонусом. Все данные передавались в реалтайме. Т.е. если бы он проебался в любой момент - данные бы остались. Алсо, в случае фейла ориентации траектория Гюйгенса дополнительно измерялась с Земли сетью VLBA с ~километровой точностью, сама по себе уникальная операция, потребовавшая установки специальных приёмников и записи 27ТБ данных. Ещё потом в Грин Бэнкс принимали сигнал, после того как Кассини скрылся за горизонтом (3.5Вт с такого расстояния!), батареи у него неожиданно долго работали, даже радиовидимости не хватило, хотя траектория была рассчитана специально.
http://www.planetary.org/blogs/emily-lakdawalla/2005/20050207-they-were-the-first-and-the-last-to-hear-from-huygens.html вот здесь есть кулстори. (http://www.planetary.org/blogs/emily-lakdawalla/2015/01151154-huygens-10-years.html алсо вот здесь эта баба пишет много интересного про посадку, про утечку данных на публику, про всю хуйню)

Кстати, быдлокодеры и так проебали запятую, Гюйгенс был запущен с фатальным багом. При программировании модуля PDRS (Probe Data Relay Subsystem), ответственного за формирование потока научных данных и его передачу на Кассини, неправильно учли влияние замедления крафта на допплеровское смещение сигнала между ним и Кассини (скорость в перицентре 5.5км/с), в результате тактирующий импульс мог проебаться и связь была бы неустойчивой. Потом в панике бегали и думали, как это пофиксить. В конце концов
Подробности есть например тут http://www.thespacereview.com/article/306/1 , охуенная статья.
Аноним 23/04/15 Чтв 07:39:27 #230 №170909 
>>170908
>В конце концов изменили маневр по сбросу лендера, и все прошло нормально (не считая проёба одного из каналов, и части фоточек вместе с ним, конечно).
Аноним 23/04/15 Чтв 10:27:35 #231 №170933 
14297740557380.jpg
14297740557421.jpg
14297740557452.jpg
14297740557503.jpg
Поясните за джеты голактеки. Я думал что они как струи воды сплошные, но наткнулся на мнение, что они внутри пустые. То есть типа они как полые конусы - по краям летят частицы а внутри почти пусто. Это правда?
Аноним 23/04/15 Чтв 12:16:18 #232 №170954 
Посмотрел кинцо про титан, верно ли я понял что человеку на его поверхности будет достаточно простого изолирующего костюма?
Аноним 23/04/15 Чтв 12:27:24 #233 №170956 
>>170864
Чото както я хуй знает. Находят же ебучих ящпегиц размеров с дом. А они даже материальной культуры не создали.
Аноним 23/04/15 Чтв 12:32:05 #234 №170957 
>>170954
Дай линк. Это первое.
Второе, я думаю нет, какое там давление? Тепмература? Радиопояс сатурна, вот это всё.

Тупой вопрос: метан горит сам по себе или только с кислородом? Говоря проще, если дать искру на Титане, он ёбнет?
Аноним 23/04/15 Чтв 12:38:23 #235 №170959 
>>170957
>Тупой вопрос
Да, действительно.
Аноним 23/04/15 Чтв 13:00:57 #236 №170963 
>>170860
>>170864
Разве гравитационное воздействие распространяеться со скоростью света?
Аноним 23/04/15 Чтв 13:15:47 #237 №170968 
>>170963
Оно же типо искривляет пространство и действует сразу
Насколько я понимаю, это никто ещё особо не знает.
Аноним 23/04/15 Чтв 13:17:35 #238 №170969 
>>170959
Для возникновения процесса горения необходимо наличие горючего вещества, окислителя и источника зажигания

Ну то есть без окислителя, короче говоря, не ёбнет?
Аноним 23/04/15 Чтв 13:17:57 #239 №170970 
>>170933
>но наткнулся на мнение, что они внутри пустые
Чо? Какие еще пустые, где, у кого такое мнение?
Аноним 23/04/15 Чтв 13:23:49 #240 №170971 
>>170957
>какое там давление?
1.5 атмосферы. Несущественно больше, чем на Земле.
>Тепмература?
Дубак-пиздец -180
>Радиопояс сатурна
Значительно меньше и слабже радиопояса Юпитера. Данные лень искать.
>Говоря проще, если дать искру на Титане, он ёбнет?
Кек, нет конечно.

Аноним 23/04/15 Чтв 14:24:05 #241 №170982 
>>170957
"Пункт назначения - Титан"
Там в основном интерьюирут тех кто создавал Кассини и управлял им.
Аноним 23/04/15 Чтв 14:27:11 #242 №170983 
>>170971
Прокатят ли лёгкие костюмы с подогревом? Или замерзнешь?
Еще не могу найти данных по уровню радиации на Титане.
Аноним 23/04/15 Чтв 14:38:55 #243 №170988 
Поясните за JWST и оче большой европейский телескоп, а конкретно про их возможности в наблюдении экзопланет. В тырнете пишут, что они даже признаки жизни на них смогут разглядеть, это правда? Если да, то что имеется ввиду под признаками жизни?
Аноним 23/04/15 Чтв 14:56:55 #244 №170992 
>>170988
>Если да, то что имеется ввиду под признаками жизни?
Спектральный анализ газовых оболочек. Т.е. атмосфер экзопланет. Больше ничего разглядеть на таких расстояниях нельзя.
Аноним 24/04/15 Птн 09:19:48 #245 №171119 
>>170983
Теоретически можно в негерметичных, если только там не воняет сильно например.
Аноним 24/04/15 Птн 09:28:01 #246 №171120 
>>170992
И каковы шансы, что он сдетектирует признаки цветущих лесов на какой-нибудь Глизе 667?
Аноним 24/04/15 Птн 09:42:05 #247 №171123 
14298577257270.jpg
>>170968
>действует сразу
Струнщики-бранщики и прочие физики-ядерщики с тобой не согласятся.

У них от этой хуйни рассыпаются все эпицикло-деференты, которые они там за пятьдесят лет насчитали.
Аноним 24/04/15 Птн 09:46:27 #248 №171125 
>>171119
>Теоретически можно в негерметичных
Нельзя, там циан есть в атмосфере. Можно отравиться. А поскольку давление 1.5 атмосферы, то холодненькое будет охуительно задувать внутрь скафандра. Холодненькое это -180 у поверхности.
Аноним 24/04/15 Птн 09:47:48 #249 №171126 
>>171120
Весьма высокие. Большое количество свободного кислорода в атмосфере - это тебе не хуй собачий.
Аноним 24/04/15 Птн 09:54:26 #250 №171128 
>>171126
А что, на ней уже и кислород обнаружили? Я думал, просто температурный режим очень хороший
Аноним 24/04/15 Птн 10:09:54 #251 №171131 
>>171128
Лол.
Я к примеру сказал, что на планете, где будет обнаружена большая концентрация молекулярного кислорода, скорее всего существуют леса, или растения с фотосинтезом. Потому как кислород в свободном виде долго существовать не будет. А не то, что где-то что-то обнаружили.
Алсо Глизе 667 - это звездная система, планеты буквами обозначают, после цифр. К примеру Глизе 667 Се. Где 667 - номер звезды в каталоге, вторая буква, если строчная - обозначение компонета, если система из нескольких светил состоит и прописная буква обозначает уже планету.
Аноним 24/04/15 Птн 10:31:31 #252 №171135 
>>171125
>там циан есть в атмосфере
У поверхности?
Аноним 24/04/15 Птн 10:41:22 #253 №171137 
>>171135
Даже если нет, сквозить будет.
" У людей, работающих в шахтах или на производствах, где в воздухе присутствуют в незначительных количествах метан и другие газообразные парафиновые углеводороды, описаны заметные сдвиги со сторонывегетативной нервной системы(положительный глазосердечный рефлекс, резко выраженная атропиноваяпроба, гипотония) из-за весьма слабого наркотического действия этих веществ, сходного с наркотическим действиемдиэтилового эфира."
Аноним 24/04/15 Птн 11:37:45 #254 №171150 
>>171135
>У поверхности?
Ага, и синильная кислота в озерах, в некотором количестве. Да и этаном дышать тоже вредно. Можно прибалдеть и пойти купаться, забыв обо всем на свете.
Аноним 24/04/15 Птн 14:52:04 #255 №171195 
>>171150
Вдохни-ка воздух при -180, ага.
Даже хотя бы просто попробуй при -40.

А тут еще и говно всякое в виде левых газов.
Аноним 24/04/15 Птн 16:23:32 #256 №171202 
>>171195
>Вдохни-ка воздух при -180, ага.
А что тут невозможного? Проблема же чисто техническая: подогрев вдыхаемого воздуха.
Много энергии не требует - выдыхаемым можно подогревать вдыхаемый. Так что можно дышать комфортно.
1,5 атмосферы - норма. Короче условия там вполне райские - даже баллоны с собой таскать не нужно, дыши в волю, сколько хочешь.
Не то что где-нибудь на луне - без баллонов сдохнешь.
Аноним 24/04/15 Птн 16:47:59 #257 №171205 
>>171150
>>171195
>>171202
Ну то есть все-таки на Титане можно разгуливать в чем-то, напоминающем гибрид костюма РХБЗ с изолирующим противогазом и снаряжения полярников, а не в неповоротливой ёбале типа той, в которой американцы ходили по Луне, верно?
Аноним 24/04/15 Птн 16:47:59 #258 №171206 
>>171202
>Короче условия там вполне райские - даже баллоны с собой таскать не нужно
Чем ты там дышать собрался? Азотом, или метаном?
Аноним 24/04/15 Птн 16:49:18 #259 №171207 
>>171205
Разгуливай хоть голым. -180 он собрался гулять в фуфайке и валенках. Дураку разве запретишь?
Аноним 24/04/15 Птн 16:56:31 #260 №171208 
14298837916000.jpg
Атмосфера в целом на 98,6 % состоит из азота
На метан приходится 1,6 % от атмосферы в целом
Имеются также следы:
- этана
- диацетилена
- метилацетилена
- цианоацетилена
- ацетилена
- пропана
- углекислого газа
- угарного газа
- циана
- гелия

Токсикоманский ледяной рай с ливнями и ледяным ураганом, лол.

>Слабая изморось из метана постоянно выпадает из облаков на поверхность, компенсируемая испарением (аналог гидрологического цикла на Земле). Выше 16 км, отделенный промежутком, лежит разреженный слой облаков из кристалликов метанового льда.

>мощные дождевые облака, хорошо заметные на фоне поверхности, быстро перемещающиеся и меняющие форму под действием ветра. Обычно они покрывают относительно небольшую площадь (менее 1 % диска), и рассеиваются за время порядка земных суток.

>Вызванные ими ливни должны быть очень интенсивными и сопровождаться ветром ураганной силы. Дождевые капли, по расчетам, достигают диаметра 1 см. Однако несмотря на то, что за несколько часов может выпасть до 25 см метана, общий уровень осадков составляет в среднем за земной год несколько см, что соответствует климату самых засушливых земных пустынь.

Че-то как-то не очень подышишь, да?
Аноним 24/04/15 Птн 17:05:59 #261 №171209 
>>171208
>Токсикоманский ледяной рай
Он был бы таким, если бы еще был бы кислород, а так смерть от удушья.
>Атмосфера в целом на 98,6 % состоит из азота
У поверхности азота около 95%, чем ближе к поверхности, тем меньше его концентрация.
Аноним 24/04/15 Птн 17:13:25 #262 №171210 
http://traditio-ru.org/wiki/Колонизация_космоса

Читай анон. И неси на обсуждение самое вкусное сюда.
Жаль, не доживем, спейсаны.
Остается только в огурцах/орбитере/SE фантазировать.
Аноним 24/04/15 Птн 17:17:42 #263 №171211 
>>171210
У нас есть терраформинг тред. Успешно тонет с ~250 постами, в районе первой-второй страницы.
Аноним 24/04/15 Птн 17:41:53 #264 №171213 
>>171211
Пошли туда, спейсаны
https://2ch.hk/spc/res/144302.html
Аноним 24/04/15 Птн 23:43:08 #265 №171292 
>>171207
При такой температуре вангую половина материалов по манде пойдут. Меховые трусы не прокатят.
Кстати, есть предположения, как правильно носить меховые трусы: мехом наружу или внутрь?
sageАноним 24/04/15 Птн 23:46:01 #266 №171293 
>>171292
Ну судя по тому что все меховые вежи носят мехом наружу, все таки наружу ящетаю.
Сука, представил и проиграл пьяного русского на Титане в шубе на голое тело.
sageАноним 24/04/15 Птн 23:49:37 #267 №171294 
14299085772770.jpg
Я не понял, Гюйгенс только одну фотку прислал с поверхности титана? Больше не нашел.
Аноним 24/04/15 Птн 23:54:50 #268 №171296 
>>171293
В наноушанке, да.
Аноним 24/04/15 Птн 23:57:12 #269 №171297 
>>171294
У вас не захватывает дух, глядя на фотке с поверхности Венеры, Марсу, Титану? Я вот лично прям прусь.
sageАноним 24/04/15 Птн 23:59:20 #270 №171298 
>>171297
Хотелось бы с супом
Аноним 25/04/15 Суб 00:01:52 #271 №171299 
>>171298
Ну скоро будет. Куриосити какой-нить накарябает на камне.
sageАноним 25/04/15 Суб 00:04:10 #272 №171300 
>>171299
>sup /spc 28.11.2027
можете скринить
Аноним 25/04/15 Суб 07:16:12 #273 №171343 
14299353725760.jpg
>>171294
Жопой искал что ли? Первый же запрос в гугле пачку выдаст. Все выкладывается в свободный доступ же. Некоторые делают вот такие визуализации: http://www.youtube.com/watch?v=sZC4u0clEc0 http://www.youtube.com/watch?v=HtYDPj6eFLc

http://atmos.nmsu.edu/PDS/data/hpdisr_0001/ все фотки с Гюйгенса (с камеры DISR) в сыром виде
http://esamultimedia.esa.int/docs/titanraw/index.htm вот здесь в чуть более удобном виде для не умеющих в исходники и тем кому не нужна привязка к остальным данным.

http://saturn.jpl.nasa.gov/photos/raw/ а здесь фотки с Кассини, за любой период
Аноним 25/04/15 Суб 07:50:09 #274 №171346 
>>171293
>>171292
Это долбоебы носят мехом наружу. Прошаренные челы носят как положено - мехом внутрь.
Аноним 25/04/15 Суб 13:26:15 #275 №171374 
>>167732
Анон, а давайте разберем по косточкам силу которая действует на меня при ускорении и торможении, что это за явление? Почему меня тянет назад при разгоне автомобиля и толкает вперед при торможении? Убьет ли меня при достижении скорости света?
sageАноним 25/04/15 Суб 13:43:22 #276 №171376 
>>171343
NYET. Я имел ввиду именно фото С поверхности.
Аноним 25/04/15 Суб 14:43:26 #277 №171383 
>>171346
Медведи и волки - долбоёбы?
Аноним 25/04/15 Суб 14:45:00 #278 №171385 
14299623006010.jpg
14299623006171.jpg
14299623006302.jpg
>>171376
Нет конечно, у него же не было механики никакой, чтобы камеру повернуть. Это тупо капсула, которая приземлилась и снимала что видела перед собой, так что в приземленном виде есть только один ракурс. Но если все же глянуть на триплеты и визуализации по ссылкам выше, увидишь что есть еще несколько фоток непосредственно перед приземлением с близкой дистанции, широкоугольным объективом DISR. Увы, высокого разрешения принципиально не вышло бы, поэтому придется полагаться на композитные реконструкции.
sageАноним 25/04/15 Суб 14:54:04 #279 №171393 
>>171383
Шуба греет (вернее сохраняет тепло) за счет плохой теплопроводнойсти воздуха между волосинками меха. Принципиально нет разницы носить наружу или внутрь, но вангую что если носить мехом внутрь он будет сваливаться и хуже выполнять функции.
sageАноним 25/04/15 Суб 14:54:52 #280 №171394 
>>171385
Неужели нельзя было прикрутить камеру с моторчиком чтобы позырить че окреть творится?
Аноним 25/04/15 Суб 16:52:22 #281 №171411 
>>171394
- 180. Масло замерзнет, механики по минимуму. Я так понимаю, никто в серьёз не рассчитывал, что зонд после посадки будет работать.
Аноним 25/04/15 Суб 17:05:14 #282 №171414 
>>171374
Это называется ускорение. Остальное можешь нагуглить.

>Убьет ли меня при достижении скорости света?
Именно ускорением? Смотря какое оно будет.
Аноним 25/04/15 Суб 17:48:55 #283 №171421 
14299733358290.jpg
>>171394
>Неужели нельзя было прикрутить камеру с моторчиком чтобы позырить че окреть творится?
Сильно усложнило бы аппарат. Там даже световодами вместо зеркал пользовались.

>>171411
>- 180. Масло замерзнет
Всякие критоксы и IKV-шные антифризные смазки здесь конечно не катят, но криогенные полиэтиленгликолевые/эфирные смазки работают даже в жидком гелии, например Apiezon N http://www.apiezon.com/products/vacuum-greases/n-grease . (хуй знает как с химической активностью, но есть изолирующие сочленения). Да и сочленения без смазки возможны. В общем это не очень просто, но вполне решаемо.

> никто в серьёз не рассчитывал, что зонд после посадки будет работать
this
Даже если почитать документы времен разработки Huygens - всё что было после посадки, декларировалось как "возможное", на работу после посадки рассчитывали как опциональный и не очень вероятный бонус. При этом планировалась она на почти любую поверхность с уклоном не более 30 градусов, включая водоёмы (на содержимое которых завезли лишь очень приблизительные ограничения до полета Кассини).
Аноним 25/04/15 Суб 19:09:56 #284 №171427 
Анон, визуальной астрономии вопрос. Есть дудка-рефрактор 90мм, реально в нее увидеть Рею? Знаю, что Титан точно можно разглядеть, но он вроде сильно ярче.
Если нет, то какая апертура нужна?
Аноним 25/04/15 Суб 20:47:47 #285 №171452 
>>171421
А что могло помешать зонду работать на поверхности?
Аноним 25/04/15 Суб 20:56:55 #286 №171456 
>>171452
Удар, зарывание, переворачивание, не-срабатывание какой-нибудь хуйни хотя бы парашюта, плохая ориентация, мало ли может произойти с АМС за миллионы километров в неизученной среде. Никто толком не знал даже что там за поверхность, догадки одни сквозь непрозрачную поверхность. Примерно как с лунными АМС, когда не знали сядет ли или утонет нахуй в лунной пыли, которая там по некоторым гипотезам была охуенно мягкой до большой глубины.
Аноним 25/04/15 Суб 23:23:43 #287 №171517 
>>171456
А вот интересно было бы посмотреть, насколько в случае с Гюйгенсом и экспедиции к Титану ДОГАДКИ совпали с ДАННЫМИ.
sageАноним 26/04/15 Вск 09:24:06 #288 №171592 
>>170316
>Прошу прощения, этот реактор - получается, одноразовый что ли?
Напомнило лазоры на ядерной энергии; еще в детстве читал околонаучную фантастику про них. Ебически жаропрочное зеркало (условно; в общем - какая-либо конструкция, формирующая луч), и ядреная боньба. Боньба взрывается, "зеркало" успевает создать когерентный пучок и тут же испаряется, луч пизохает по клятым америкосам (например, с орбиты). Такая вот вундервафля. Одноразовый лазор нивираятной мощи.
Аноним 26/04/15 Вск 09:27:41 #289 №171593 
>>170441
>Да. Локхид сделал уже.
Вот, кстати, меня всегда мучал вопрос: а как они выделяющееся тепло с него снимать собираются? Там ведь воду не прокачаешь через активную зону.
Аноним 26/04/15 Вск 11:02:10 #290 №171598 
>>171593
Оно будет уходить в варп
Аноним 26/04/15 Вск 11:08:36 #291 №171600 
>>171593
Так там небось сатаниские элементы преобразующие тепло в пестричество.
Аноним 26/04/15 Вск 12:15:39 #292 №171627 
>>171593
они както должны выводить продукты реакции, которые тоже представляют собой плазму которую ты можешь пульнуть в паровой котел к примеру или МГД. перенос энергии излучением тоже никто не отменял, магнитное поле удерживающее плазму не будет влиять на излучение также как на саму плазму. вообщем только успевай в паровой котел запихивать
Аноним 26/04/15 Вск 13:06:44 #293 №171659 
Skylon, распил бабок или годный проект?
Аноним 26/04/15 Вск 13:11:15 #294 №171663 
>>171659
Скандалы, интриги, расследования.
Аноним 26/04/15 Вск 13:19:24 #295 №171670 
>>171659
>проект
Тхис. Почему о них нихуя не слышно? Ещё живы?
Аноним 26/04/15 Вск 13:45:21 #296 №171678 
>>171427
http://kosmoved.ru/raschet_teleskopa.shtml

>Предельная звёздная величина (m)
>m=2.1+5lg(D), где D – диаметр телескопа в мм

2.1+5lg(90)=11.87
Рея сейчас величиной 10.34, значит можешь ее увидеть в виде оче слабой звездочки, эквивалентной 5m без скопа.
Аноним 27/04/15 Пнд 01:21:03 #297 №171866 
>>171678
Напомнил.
Как узнать какую максимальную звездную величину может человеке невооруженным глазом с проблемами со зрением видеть?
Аноним 27/04/15 Пнд 01:30:40 #298 №171867 
>>171866
Как бэ проблемы со зрением бывают очень разными.
Аноним 27/04/15 Пнд 02:03:37 #299 №171870 
>>171867
Предположим близорукость 0,6-0,7
Аноним 27/04/15 Пнд 03:13:26 #300 №171876 
На орбите какой планеты солнечные панели способны выдать сколь-нибудь значимую мощность(порядка сотни ватт)?
sageАноним 27/04/15 Пнд 03:21:59 #301 №171878 
>>171876
КПД панелей гуглишь, энергию на квм солнечного света гуглишь.
Аноним 27/04/15 Пнд 03:23:56 #302 №171879 
Атмосфера Титана почти полностью (98,4%) состоит из азота. Значит там небо голубое, как на земле?
Аноним 27/04/15 Пнд 03:27:37 #303 №171880 
>>171876
Простите, а какой площади подразумеваются солнечные панели? Поток солнечной энергии обратно пропорционален квадрату расстояния от Солнца, так что за орбитой Марса уже чет не очень юзать панельки.
Аноним 27/04/15 Пнд 03:29:36 #304 №171881 
>>171880
Скажем, три квадратных километра
Аноним 27/04/15 Пнд 04:07:22 #305 №171882 
>>171879
Небо там желтое. В основном из-за углеводородов в атмосфере. Учитывая плотность, с поверхности Титана даже Сатурна скорее всего не видно.
Аноним 27/04/15 Пнд 07:49:56 #306 №171893 
>>171881
Почему не 30? Охуеть.
Аноним 27/04/15 Пнд 08:10:44 #307 №171894 
14301114446070.jpg
14301114446081.jpg
14301114446102.jpg
14301114446123.jpg
>>171879
В разных слоях по-разному. Там очень разный состав и плотность, в верхних слоях вообще постоянная химическая реакция под действием света идёт, по сути Титан это большая тлеющая свечка. Вот панорама на основе фоток спуска на 110, 70 и 25км, скорректированная по данным инструментов - в общем наиболее достоверное представление видимости в разных слоях, которое может быть. Здоровенное "солнце" это Сатурн, на самом деле, его видно только в верхних слоях. Интересно, что подумали бы криогенные кербалы на Титане, впервые запустив ракету и увидев этот шар ебаный.

4 - фотка слоев атмосферы с максимально близкого пролёта Кассини в 850км, ниже там уже аэроторможение получится
Аноним 27/04/15 Пнд 08:36:50 #308 №171897 
>>171882
Но ведь метана всего полтора процента
Аноним 27/04/15 Пнд 08:48:53 #309 №171898 
>>171894
Там может и будут кербалы лет через лярд. Что там насчёт радиации, Гюйгенс менял? Счётчика гейгера у него вроде не было.
Аноним 27/04/15 Пнд 09:33:22 #310 №171901 
>>171897
>Но ведь метана всего полтора процента
И что? Зато атмосфера 4 раза толще и плотнее.
Аноним 27/04/15 Пнд 12:38:30 #311 №171922 
>>171897
Ну в газе из плиты, например, лишь десятисячные доли процента этилмеркаптана. Это не мешает этому самому газу пахнуть исключительно одорантом.
Аноним 27/04/15 Пнд 14:26:03 #312 №171940 
А если накачать атмосферу титана кислородом, ебнет? Сколько процентов кислорода надо для взрыва метана при 1.5% его содержании в воздухе? Для земных условий 5% смесь метан-кислород ебашит
Аноним 27/04/15 Пнд 19:46:37 #313 №172057 
>>171894
Че-то у меня очко сжалось от фоток. Сильнее только от Венеры сжимается.
Аноним 27/04/15 Пнд 19:48:15 #314 №172058 
>>171940
Давай лучше водород подорвем на Юпитере.
Аноним 27/04/15 Пнд 20:12:18 #315 №172070 
>>172058
А давай. Как там запустить цепную термоядерную реакцию?
Аноним 27/04/15 Пнд 20:41:47 #316 №172107 
>>172070
Нужно увеличить массу. Думаю, полтора триллиона черных монолитов должно хватить.
Аноним 27/04/15 Пнд 21:00:27 #317 №172136 
>>172058
Металлический водород не горит же?
Аноним 27/04/15 Пнд 21:04:04 #318 №172140 
>>172136
Если в соединении с металлическим кислородом-вполне
Аноним 27/04/15 Пнд 23:14:08 #319 №172279 
>>172057
И у меня. Но в хорошем смысле.
Аноним 27/04/15 Пнд 23:14:50 #320 №172280 
>>172057
Кстати есть приличные фотки с Венеры? То, что я видел, делали советские исследовательские аппараты, но они так, не супер-супер.
Аноним 28/04/15 Втр 00:10:12 #321 №172323 
>>172280
>90 атмосфер
>500 градусов
А че ты хотел?
Аноним 28/04/15 Втр 01:32:41 #322 №172364 
>>172280
Кроме советских никто и не делал. А приличных мы вряд-ли скоро увидим, потому-что на Венеру почти всем начхать.
Аноним 28/04/15 Втр 01:44:23 #323 №172368 
>>172364
Слишком сложно исследовать, даже зонд с йоба-теплозащитой не продержится больше пары часов.
Интересно, почему на Меркурии не сажали зонды, там возле полюса вполне приемлемый температурный режим, предположительно лед даже есть. Слишком много дельты надо?
Аноним 28/04/15 Втр 01:54:57 #324 №172373 
Хочу услышать критическое мнение о венерианском "скорпионе". Там ведь действительно похоже на уползающее раненое животное. Не мог же камень сам переместится на десятки сантиметров.
Аноним 28/04/15 Втр 01:58:37 #325 №172375 
>>167732
У мя 2 вопроса:
1. Насколько лекго или нелегко подделать лунный грунт на Земле.
2. Сколько может стоить прибор-идентификатор лунного грунта, если он вообще нужен?
Аноним 28/04/15 Втр 01:59:21 #326 №172376 
>>172373
А банальный вариант с тем, что это мог бы быть отвалившийся от зонда кусок говна, переносимый ветром, не покатит (как недостаточно уфологичный)?
Аноним 28/04/15 Втр 02:04:03 #327 №172379 
14301758439370.jpg
>>172376
Допускаю. Но согласно исследованиям венерианская атмосфера лишена существенных ветров. Да и что тут могло оторваться я хз. I want to believe короче, Скалли.
Аноним 28/04/15 Втр 02:08:37 #328 №172381 
14301761171710.jpg
>>172379
Уж не знаю, какие ты тым исследования смотрел, но ветра там 10м/с у поверхности. Какую-нибудь мелочёвку перенести хватит вполне.
Аноним 28/04/15 Втр 02:09:04 #329 №172382 
14301761441480.png
>>172375
Предвижу луносрач и сразу говорю - Сеня, не надо.
Советские станции привозили немного грунта минимум дважды. Можно сравнить с американскими.
Были они там, были, хватит уже.
Аноним 28/04/15 Втр 02:09:50 #330 №172383 
>>172381
Да? Ну может устаревшие данные читал. Ладно, проехали.
Аноним 28/04/15 Втр 05:49:22 #331 №172414 
>>172070
>>172107
да при текущей массе цепная реакция вроде как хуй, но есть вариант
надо обжать, обкладываем метровым слоем антивещества, сверху присыпаем метров 10-50 вещества, чтобы коммулятивный эффект был для нашего антивещества, ну вообщем ты понел принцип
интересно сколько антивещества по толщине слоя надо на такую схему (допустив что это к примеру будет антижелезо)
чето у меня один nan в моей виртуальной симуционной машине крутится - не пойму толи метр это оверкил или не хватит.
обжать дейтерием метровым слоем разогнанным до 0.9с - не хватитает по ходу, но походу можно отстрелить щедрый слой вещества с планетки, ядру тоже достанется,
надо чтобы ударная волна была достаточной для начала реакции гдето возле ядра, тогда как усилитель получится, и можно будет некисло прибарахлиться веществом.
Аноним 28/04/15 Втр 12:08:08 #332 №172476 
>>172382
Причем тут были-не были. Ни разу не был инициатором луносрачей. А вот твой пост как бы намекает.

Меня интересует только лишь возможность подделки лунного грунта. Если я пойду и накопаю в песочнице кошачьих какашек, прилеплю сертификат и скажу на ебее, что открывать колбу зогпрещено во избежание, как скоро я буду раскрыт? Легенда о том, что в годы развала чего только не происходило в советских музеях и бывших шарашках - прилагается.
Аноним 28/04/15 Втр 12:12:40 #333 №172478 
Ученые проводили испытания на земном лунном грунте. Хотя бы в том конкурсе на луноходы.
Аноним 28/04/15 Втр 12:14:42 #334 №172479 
https://ru.wikipedia.org/wiki/Реголит
Смешиваешь что там написано и упарываешь. Можно без кошачих какашек
Аноним 28/04/15 Втр 12:22:03 #335 №172485 
>>172476
Подделать можно что угодно. Любой специалист раскусит подделку с помощью микроскопа, даже анализ изотопов не понадобится. Ключевая разница в форме микрочастиц. На Земле грунт имеет окатистую форму или края. Это связано с эрозией, воздействием воды и воздуха. На Луне такого нет нихуя.
Аноним 28/04/15 Втр 17:28:21 #336 №172625 
Ананасы, поясните за реальность. (В треде огурцов ksp меня проигнорили).
Посмотрел тут видос утреннего вывода Прогресса и по видео кажется что он почти сразу берет угол в 40-45°, так ли это? Как ирл вообще это делают?
https://www.youtube.com/watch?v=BJhSabDSohI
Аноним 28/04/15 Втр 17:51:21 #337 №172629 
>>172625
Линейная скорость вращения Земли (на экваторе) — 465,1013 м/с (1674,365 км/ч)
Аноним 28/04/15 Втр 19:10:32 #338 №172661 
14302374327490.jpg
14302374327601.jpg
>>172625
Все верно тебе кажется.
> Как ирл вообще это делают?
В разных миссиях по-разному. До МКС из Байконура по "укороченной" шестичасовой схеме (по которой летел Прогресс-М-27М) РН Союз обычно летит практически по диагонали прямо со старта, да.
Аноним 28/04/15 Втр 20:11:57 #339 №172700 
>>172625
А еще, просто-таки напросто таки видно по следу, что что-то пошло не так, и извращаться он начал уже километров с сорока, я так думаю.
Аноним 28/04/15 Втр 20:36:21 #340 №172714 
>>172661
Блин, а ведь эти куски ракеты падают назад на поверхность планеты. Неужели для каждого рассчитывают место падения, чтобы случайно в какой-нибудь небоскреб в мегаполисе не залетел? Или на парашютах спускают?
Аноним 28/04/15 Втр 20:41:44 #341 №172716 
14302429046210.jpg
14302429054351.jpg
14302429054432.jpg
>>172714
Есть специальные зоны падения, где никого не бывает на десятки-сотни км, и их объявляют опасной зоной.
Аноним 28/04/15 Втр 20:57:05 #342 №172725 
>>172716
Ну а если забредет кто и его прибьет? Большая компенсация будет?
Аноним 28/04/15 Втр 21:07:31 #343 №172730 
>>172716
А с Канаверала как? В окиян?
Аноним 28/04/15 Втр 21:23:55 #344 №172746 
>>172725
>Ну а если забредет кто и его прибьет? Большая компенсация будет?
Премия Дарвина будет.
Аноним 28/04/15 Втр 21:26:52 #345 №172748 
>>172725
Да даже если упадет и развалит сарай, и даже видево попадет в инторнет, ничего не будет.
Аноним 28/04/15 Втр 21:29:00 #346 №172751 
>>172748
Как так? А журнашлюхи кок-пок-пок-ку-кудах
Аноним 28/04/15 Втр 21:40:43 #347 №172755 
>>172751
>А журнашлюхи кок-пок-пок-ку-кудах
За все время ни одного сюжета про то, как кого-то уебало ракетой по голове.

>Алгоритм выполнения мероприятий по обеспечению безопасности в районах падения ОЧ РН включает в себя:
>заблаговременное оповещение местных администраций, на территории которых находятся районы падения ОЧ РН, о предстоящем пуске РН.

>Предпусковое обследование территории РП с использованием наземного транспорта или авиасредств.

>Эвакуация в безопасный район людей (охотников, рыболовов, сборщиков ягод, ученых, туристов и других лиц), обнаруженных в РП, наблюдение за пуском РН (падением ОЧ);
Аноним 28/04/15 Втр 21:54:06 #348 №172761 
>>172755
А если спрячется кто-то? И его уебет куском движка c автографом Комарова наглухо?
Аноним 28/04/15 Втр 22:08:08 #349 №172768 
>>172761
Дебил пошел гулять в зону падения, несмотря на все предупреждения. Дебил спрятался от тех, кто вынимает долбоебов из зоны возможного падения. Дебила прибила ракета нахуй. Дебила нашли придавленного куском ракеты.
Что по-твоему должно быть? Ну поминки родственники организуют. Кто-нибудь встанет, поднимет стопку и скажет: «Жил как мудак и сдох как мудак, ну и хуй с ним».
Аноним 28/04/15 Втр 22:11:11 #350 №172769 
14302482720250.png
>>171867
> Земля (при наблюдении с Солнца)
Аноним 28/04/15 Втр 22:24:27 #351 №172775 
>>172768
NYET. Роскосмос засудят на лярд баксов. Он вообще может лунатик был или сумасшедший, что тогда?
Аноним 28/04/15 Втр 22:38:43 #352 №172784 
14302499239840.jpg
Посоны, а до какого момента АМС Мессенжер будет, при сближении с Меркурием, передавать фотографии на землю? Вплоть до столкновения? Или он уже ВСЕ в спящем режиме работает?
Аноним 28/04/15 Втр 22:43:52 #353 №172790 
Осне тупой вопрос. Почему на Гюйгенс нельзя было поставить РИТЭГ и более мощную антенну, чтобы можно было замутить длительное исследование?
Аноним 28/04/15 Втр 22:45:03 #354 №172793 
>>172775
>Роскосмос засудят на лярд баксов. Он вообще может лунатик был или сумасшедший, что тогда?

Ты сразу бы сказал, что ты ебанутый на всю голову.

Аноним 28/04/15 Втр 22:50:17 #355 №172798 
>>172793
А я тут при чем? Вот разве это правильно что человека убьет куском ебучей ракеты? Даже если он даун.ты просто человеконенавистник какой-то
Аноним 28/04/15 Втр 22:57:55 #356 №172799 
14302510759540.jpg
>>172784
Вот этот снимок получен 26 апреля. Значит что-то еще передает.
Аноним 28/04/15 Втр 23:07:29 #357 №172803 
>>172790
Потому что больше нахуй не надо. Все задачи зонд выполнил.

>The power system for the Huygens probe was selected on the basis of the mission requirements and to fit the probe's mass, cost, telecommunications and other constraints. A battery was determined to be the right power source to meet the mission requirements. The scientific objectives of the Huygens probe mission are to characterize the chemical reactions occurring in the Titan atmosphere, find the source of abundant methane in the atmosphere, determine if there are seas or lakes on the surface, and determine if more complex organic compounds and pre-biotic molecules exist on Titan. The probe was designed and instruments selected to answer these high-priority questions. The battery is designed to provide 3 hours of electrical power - more than adequate to meet the probe's electrical needs in the 2 and 1/2 hour descent. In that time, the probe should return a great wealth of observations and measurements that answer the science objectives and leave us with new questions that could be addressed by future missions. For longer-lived landed or orbital missions to Titan in the future, radioisotope power sources, and/or a space nuclear reactor under study by NASA's Prometheus program, would be among the power sources considered.
http://saturn.jpl.nasa.gov/faq/FAQHuygens/
Аноним 28/04/15 Втр 23:16:50 #358 №172813 
>>172798
>Вот разве это правильно что человека убьет куском ебучей ракеты?
Если он в обход всех кордонов/предупреждений/людей попёрся туда, где ему может ёбнутся кусок ракеты на голову, то он это заслужил. Ты же не полезешь на минное поле/стрельбище в здравом уме, если там будет куча предупреждающих знаков и людей, уводящих тебя?

мимовылезизридонли
Аноним 28/04/15 Втр 23:21:55 #359 №172815 
>>172813
ой не пизди что там вся территория огорожена и знаками утыкана, это невозможно
Аноним 28/04/15 Втр 23:43:45 #360 №172823 
Можно ли нелегально слетать в космос? Например ночью скрытно проникнуть (или подкупив охрану) спрятаться в грузовике отправляющемся к МКС? А когда космонавты начнут разгружать ящики выскочить в криком "Арпеджио!!!". Вот сюрприз будет.
Возможно такое провернуть?
Аноним 29/04/15 Срд 00:18:32 #361 №172845 
>>172823
В грузовик через форточку залезешь? Без системы жизнеобеспечения да, сюрприз будет тот еще.
Аноним 29/04/15 Срд 00:27:03 #362 №172847 
14302564238900.jpg
>>172845
Ну как-то же туда грузы заносят.
Я все продумал. Смотри, чтобы не лопнуть можно одется в такой костюм как пикрелейтед.Ну само собой взять кислородных баллонов побольше и тепло одеться, в термобелье например.
sageАноним 29/04/15 Срд 00:30:57 #363 №172852 
>>172845
Нахуй ты кормишь этого полудурка?
Аноним 29/04/15 Срд 00:46:47 #364 №172859 
>>172852
Разве тред не для эитого создан?
Аноним 29/04/15 Срд 01:09:56 #365 №172868 
>>172815
>>172815
http://newsland.com/news/detail/id/1525276/
http://vestigator.info/forum/index.php?topic=394.0
http://www.5-tv.ru/news/95509/

мимо >>172625 -кун
Аноним 29/04/15 Срд 01:29:46 #366 №172876 
14302601864570.jpg
>>172847
Был же недавно тред, МКС зогхватить хотели.
Но не помню я, чем дело кончили.
sageАноним 29/04/15 Срд 01:32:21 #367 №172877 
>>172876
>Но не помню я, чем дело кончили.
Омичей-полуёбков отпустило и тред затонул со всем экипажем.
Аноним 29/04/15 Срд 14:18:48 #368 №173016 
>>172868
> в регион поступило оборудование для обследования людей на молекулярном уровне.
чето срзу стартрек вспомнился
Аноним 29/04/15 Срд 14:22:03 #369 №173017 
>>173016
Биореакторы привезли. Если жалуетесь на падающие ракеты с гептилом, проходите внутрь, ложитесь, расслабьтесь.
Аноним 29/04/15 Срд 14:33:04 #370 №173018 
Почему аудиозапись с Венеры есть, а с Марса нет?
Аноним 29/04/15 Срд 14:35:08 #371 №173020 
>>173018
Потому там нечего записывать
Аноним 29/04/15 Срд 14:36:27 #372 №173022 
>>173018
А где можно послушать звуки Венеры? Гугл ломается на таком запросе
Аноним 29/04/15 Срд 15:06:07 #373 №173046 
>>173022
https://www.youtube.com/watch?v=TzP3jwZod5Y
Аноним 29/04/15 Срд 15:20:54 #374 №173048 
>>173046
Это из разных кусков запись? Или порывы ветра?
Аноним 29/04/15 Срд 15:38:06 #375 №173053 
>>173016
>чето срзу стартрек вспомнился
Готовься вспоминать всё чаще.
Аноним 29/04/15 Срд 16:31:55 #376 №173081 
>>173053
надеюсь на это, бро, я надеюсь
Аноним 29/04/15 Срд 16:39:23 #377 №173086 
>>173020
Нет уж, там не так пусто, как в твоей голове, и полным полно звуков.
Аноним 29/04/15 Срд 17:07:20 #378 №173092 
>>173086
>маняоскорбления

Лол.

Там плотность воздуха слишком низкая, чтобы звук нормально распространялся, очевидно. Хотя где-то была запись колёс кьюриосити стучащих по камням
Аноним 29/04/15 Срд 18:54:17 #379 №173126 
14303228573800.png
>>173048
Один кусок, вначале слышно ветер и гейзер пук твоей мамаши, а потом какая-то дрель.
http://www.youtube.com/watch?v=krt9RjAL50Q

А вот тебе запись гроз http://scienceandtec.ucoz.ru/_ld/0/15_V-11_COMPL-AMOU.mp3

И вот тебе ссылоки:
http://scienceandtec.ucoz.ru/forum/3-3-1
http://geektimes.ru/post/228199/
http://old.computerra.ru/vision/576546/
http://nashavselenaya.blogspot.ru/p/blog-page_24.html

http://www.yaplakal.com/forum2/topic296841.html
Аноним 29/04/15 Срд 19:10:38 #380 №173132 
>>173126
Но кто на Венере может работать перфоратором?
Аноним 29/04/15 Срд 20:18:57 #381 №173160 
>>173132
очевидно же - человек-сосед
 Sasha 29/04/15 Срд 22:19:45 #382 №173234 
Поясните за излучение Хоккинга. Вот на границе горизонта событий в вакууме происходит флуктуация, античастица улетает в черную дыру, аннигилируя в ней часть массы, а нормальная частица улетает в пространство. Почему по теории вероятности 50% на 50% в черную дыру не улетают обычные частицы? Т.е., это излучение Хоккинга типа испаряет черную дыру, но почему, блжад?
Аноним 30/04/15 Чтв 00:57:57 #383 №173312 
>>173234
почитай еще https://ru.wikipedia.org/wiki/Эффект_Унру
частица отрицательной энергии, в википедии, не означает античастица. те ящитаю норм, фабрика по производству антивещества будет работать
Аноним 30/04/15 Чтв 10:45:07 #384 №173423 
>>173312
Изучил, все стало еще более запутано. Почему же тогда антивещество должно именно уменьшать массу черной дыры, а не уменьшать, наоборот, массу остальной части Вселенной?
Аноним 30/04/15 Чтв 12:33:07 #385 №173464 
>>173234
>>173423
Суть такова: рождается пара частица-античастица, но какая из них проваливается в за горизонт событий - неважно, так как одна провалилась, а вторая доступна для наблюдения. Значит, согласно закону сохранения энергии, провалившаяся частица обладает отрицательной энергией.
Этот процесс можно представить как заём энергии вакуумом у внешнего поля для рождения пары частица-античастица. В отсутствие чёрной дыры аннигиляция «возвращает» энергию полю. При наличии чёрной дыры аннигиляции не происходит, одна из частиц улетает к наблюдателю, унося часть «занятой» энергии, тем самым уменьшая энергию, и, следовательно, массу чёрной дыры.
Аноним 30/04/15 Чтв 13:32:59 #386 №173478 
Что произойдет если самая легкая чд встретится с самой тяжелой звездой?
Аноним 30/04/15 Чтв 13:48:34 #387 №173487 
>>173478
http://www.youtube.com/watch?v=xFLU0xId9k8
Аноним 30/04/15 Чтв 13:48:55 #388 №173489 
>>173478
Самая легкая ЧД - это понятие очень растяжимое. Если имеется ввиду планковские черные дыры, то они испаряются практически моментально.
Если имеется ввиду самые легкие черные дыры звездной массы, то все зависит от угла и скорости встречи, ну и расстояния. В обычно черная дыра просто перетягивает вещество звезды на себя.
https://www.nasa.gov/mission_pages/chandra/news/H-12-056.html#.VUIIWCHtlBc
Аноним 30/04/15 Чтв 14:35:46 #389 №173500 
>>173464
>Значит, согласно закону сохранения энергии, провалившаяся частица обладает отрицательной энергией. Анон, можешь на пальцах объяснить интерпретацию закона сохранения энергии для этого случая? Почему бы черной дыре наоборот не взять себе обычную частицу с положительной энергией?

Вот тут опять загвоздка. Почему именно отрицательной? Проваливается же в общем случае рандомная частица. Кажется, я подбираюсь к сути своего непонимания.
Аноним 30/04/15 Чтв 15:17:50 #390 №173509 
>>173500
https://cosmos.d3.ru/comments/474322/
На пальцах =)

На выходе всегда получается частица с положительной энергией потому что она существует. А та, которая провалилась - не существует. Поэтому на выходе мы имеем наблюдаемое/измеряемое излучение.
Аноним 30/04/15 Чтв 15:33:43 #391 №173515 
>>173509

Из статьи:
>А Хокинг нудит что–то маловразумительное — одна частица была с отрицательной энергией и она упала в дыру, другая, которая оказалась с положительной, улетела прочь… только в непонятки народ вводит.

Оказывается, для того, чтобы создать устойчивую пару частица-античастица (а для устойчивости их надо разнести в прострастве), надо затратить энергию. Горизонт черной дыр забирает одну и частиц, придавая ей кинетическую энергию. Т.е., за весь это "маскарад" платит черная дыра, из-за чего она теряет свою энергию, что эквивалентно потере массы
Я все правильно понял?
Аноним 30/04/15 Чтв 15:37:33 #392 №173516 
>>173515
Натюрлих, камераден
Аноним 30/04/15 Чтв 16:02:27 #393 №173528 
14303989479740.jpg
>>173509
Вот же крутой чувак это. Натыкаюсь на разных сайтах - пишет как ёбаный господь.
Аноним 30/04/15 Чтв 17:38:53 #394 №173562 
14304047338230.gif
>>173528
и жжот как сотона?
sageАноним 30/04/15 Чтв 17:44:38 #395 №173564 
>>173487
Почему-то вспомнил историю друга о том, как он трахался с жирухой.
Аноним 30/04/15 Чтв 22:40:39 #396 №173662 
>>167732
Что произойдет, если столкнутся две черные дыры?
Аноним 30/04/15 Чтв 22:46:32 #397 №173667 
>>173662
Они сольются в одну.
Аноним 01/05/15 Птн 09:35:17 #398 №173795 
>>173662
Вопрос из разряда "а Бог есть?".
Аноним 01/05/15 Птн 10:17:25 #399 №173800 
>>173795
Нихуя подобного.
Аноним 01/05/15 Птн 10:56:58 #400 №173805 
>>173667
Слив будет защитан?
Аноним 01/05/15 Птн 13:19:49 #401 №173853 
Самые нубские вопросы, поясните совсем незнающему и по хардкору. Почему только Россия может доставлять разные штуки на мкс, почему шатл перестали выпускать в космос. Написано, что Россия единственная имеет управляемые корабли, а у остальных что неуправляемые ?
Аноним 01/05/15 Птн 13:49:19 #402 №173867 
>>173853
>Почему только Россия может доставлять разные штуки на мкс
Ну не только Россия. Грузы доставляли европейские грузовики ATV, раньше шаттлы тягали, теперь вот Дракон таскает, несколько ходок делал японский HTV, возможно когда-нибудь взлетит Сигнус.
>почему шатл перестали выпускать в космос
Потому что очень дорого. Для грузовика - он слишком дорогой, равно как и для ротации экипажей. После постройки МКС у шатлов не осталось работы.
>Написано, что Россия единственная имеет управляемые корабли, а у остальных что неуправляемые ?
Где это написано?
У России корабли могут в автоматическую стыковку. Впрочем автоматическая стыковка была успешно реализована на европейском ATV.

Аноним 01/05/15 Птн 14:10:18 #403 №173875 
14304786186680.jpg
>>173853
Ты начитался журналамерской хуйни. Речь о пилотируемых кораблях с людьми на борту. Их было два - Shuttle и Союз, по замыслу они обеспечивали поддержку друг друга (приостановят полёты одного - будет летать другой).

Шаттл это очень универсальная, огромная и дорогая йоба, поэтому от него отказались в 2011, когда у него совсем не стало задач кроме тягания экипажа (раньше он строил станцию). Союз - это маленькая капсула на три человека типа Аполлоновских модулей, выводимая маленькой ракетой, к тому же она может оставаться как спасательная шлюпка на МКС, поэтому её юзают как основное средство.

А грузовики, которые космонавтам жрачку, оборудование и эксперименты возят - их много разных. Правда опять в конце концов остались только Прогрессы и Dragon в основном, остальные по разным причинам вывели из эксплуатации.
Аноним 01/05/15 Птн 15:00:04 #404 №173903 
>>173867
>когда-нибудь взлетит Сигнус
Уже давно.

Алсо поясните за европейцев и японцев: чому так мало? Впилили кучу денег на разработку, чтобы слетать пару раз?
ATV вот вообще дикий вин, имхо.
Аноним 01/05/15 Птн 15:40:10 #405 №173929 
>>173903
Про японцев не скажу, но европейцы на базе ATV пилят служебный модуль Ориона, например.
Аноним 01/05/15 Птн 16:18:46 #406 №173943 
>>173929
Он будет юзаться до допила американского.
Алсо это никак не меняет того факта, что с виду очень годный транспортник ATV не летает. Чому так?
Аноним 01/05/15 Птн 21:09:00 #407 №174145 
>>173903
>Уже давно.
Сигнус был только в тестовых полетах. Говорить о нем, как о состоявшемся грузовике рановато, ящщитаю.
>ATV не летает.
Не летает, потому что дорого. У каждого участника на МКС свой вклад, в денежном эквиваленте.
Аноним 01/05/15 Птн 21:34:19 #408 №174169 
>10122 протонов понадобится, чтобы набить Обозримую Вселенную под завязку, плотненько так, протончик к протончику, впритык.

И тут меня накрыло. Что будет то если так сделать? Бабах?
Аноним 01/05/15 Птн 22:33:54 #409 №174212 
>>174169
>Что будет то если так сделать?
Кисель будет. Представь, вся обозримая вселенная - кисель.
Аноним 01/05/15 Птн 23:31:49 #410 №174242 
>>174169
>Что будет то если так сделать? Бабах?
При малейшей неоднородности заполнения случится гравитационный коллапс вся эта поебень начнется обваливаться вновь испеченную ЧД.
Мне так с дивана видится.
Аноним 01/05/15 Птн 23:43:02 #411 №174248 
>>174169
Протоны быстро охладят эфир и растают до менее энергоемких структур.
Аноним 01/05/15 Птн 23:52:36 #412 №174252 
>>174248
Ацюковский, больше не принимайте те таблетки, которые вы нашли у внука.
Аноним 02/05/15 Суб 04:08:26 #413 №174370 
Поясните за вояджеры. Если сейчас склепать аппарат со схожими задачами, даст ли профит использование на нем ионников?
Аноним 02/05/15 Суб 12:23:32 #414 №174475 
>>173489
Вроде бы звезда должна притягивать и распидорашивать чд же нет? Гравитация = масса так ведь?
Аноним 02/05/15 Суб 12:44:58 #415 №174482 
>>174475
Нет, потому что гравитационное взаимодействие обратно пропорционально квадрату расстояния, между объектами. Условно говоря, радиус ЧД в 5 солнечных масс, будет около 15 км. Тогда как самая большая звезда, имея массу в 120 солнечных, будет иметь радиус в 240 радиусов Солнца. Это огромная хуйня, с весьма разряженной внешней оболочкой. Поэтому ЧД запросто перетягивает вещество с компаньона.
Аноним 02/05/15 Суб 13:04:46 #416 №174487 
>>174370
>даст ли профит использование на нем ионников?
Безусловно даст. Ионники Dawn, например, способны прибавлять почти 100 км\ч каждые 4 дня.
Аноним 02/05/15 Суб 13:42:13 #417 №174498 
Как-то читал про чувака который в 70-х решил делать максимально дешёвые ракеты из говна и палок, доставляющие десятки тонн на орбиту. Кто-нибудь знает как его звали? Помню что он был миллионером, а его программу закрыли под давлением лобби Боинга в НАСА.
Аноним 02/05/15 Суб 14:40:04 #418 №174535 
>>173487
>чтобы не утомлять никого
Бесят люди, которые довольны собственным невежеством.
Аноним 02/05/15 Суб 15:59:29 #419 №174611 
>>174487
>почти 100 км\ч
Как-то некосмически написал. Примерно 25 м/с.
Аноним 02/05/15 Суб 16:02:17 #420 №174613 
>>174498
Не про OTRAG ли ты? Детали не совпадают, правда.
sageАноним 02/05/15 Суб 16:08:11 #421 №174617 
>>174370
http://en.wikipedia.org/wiki/Interstellar_probe
Вот только ненужно никому.
Аноним 02/05/15 Суб 16:44:19 #422 №174637 
>>174613
Не. Блин не могу найти. Помню что он собирался запускать с морской платформы (и вроде запустил) ракеты жрущие дохера топлива но осне дешёвые в изготовлении.
Аноним 02/05/15 Суб 16:50:25 #423 №174641 
Какова вероятность того что горизонты обнаружить на Плутоне останки внеземной цивилизации?
sageАноним 02/05/15 Суб 17:01:54 #424 №174647 
>>174641
Отрежь себе член.
Аноним 02/05/15 Суб 17:04:57 #425 №174648 
>>174647
Как это поможет в обнаружении внеземных цивилизаций?
sageАноним 02/05/15 Суб 17:09:49 #426 №174653 
>>174648
Ты избавишь человечество от своих генов, таким образом помогая эволюции (что важно, ибо в сегодняшнем мире выживают и плодятся все, а не только самые умные). Будущие поколения будут становится всё совершеннее, и в итоге встретят внеземную цивилизацию хорошо подготовленными.
sageАноним 02/05/15 Суб 17:11:16 #427 №174655 
>>174653
>становиться
Аноним 02/05/15 Суб 17:13:57 #428 №174658 
>>174653
Ловко ты его обосрал.
Аноним 02/05/15 Суб 17:35:12 #429 №174661 
лил
Аноним 03/05/15 Вск 00:19:26 #430 №174833 
Посоветуйте годных псевдодокументальных фильмов про космические путешествия. Можно про путешествия в недалеком будущем, но чтобы без фантастики. Например вот это понравилось https://www.youtube.com/watch?v=c3dejx_P-x0
Аноним 03/05/15 Вск 02:44:02 #431 №174869 
>>174833
Europa Report, например. А вообще тут где-то фильмотред был.
Или тебе совсем прям документально-популяризаторский стиль нужен?
Аноним 03/05/15 Вск 03:07:01 #432 №174873 
>>174869
Ну чтобы интересно смотреть было. Европа репорт зашибись кстати.
sageАноним 03/05/15 Вск 04:14:53 #433 №174884 
>>174873
Вообще из головы разве что обе Космические Одиссеи всплывают, ещё Контакт, или как-то так, про то как SETI словила сигнал пришельцев и уговорила всех построить ЙОБУ. Были интересные документалки 2000-х от Дисковери про отправку зондов к одной из ближайших звёзд в середине века.
Вообще могу ещё завтра пару книг кинуть, которые почитать всё собираюсь, на тему реалистичного полёта к Марсу. И фильманов подкину, просто надо бы с ПеКи посмотреть, а я с телефона сижу.
sageАноним 03/05/15 Вск 04:15:55 #434 №174885 
>>174884
*на ПеКе
Аноним 03/05/15 Вск 11:09:14 #435 №174924 
>>174884
Было бы неплохо
Аноним 03/05/15 Вск 15:07:37 #436 №174981 
>>170988
>Если да, то что имеется ввиду под признаками жизни?
однако какой осторожный и вдумчивый кун.
респект
Аноним 03/05/15 Вск 15:51:08 #437 №175010 
>>170988
Ядерные взрывы, тепловое излучение, вонь, эманации страха вот это все
Аноним 03/05/15 Вск 16:04:39 #438 №175017 
14306582795340.jpg
>>175010
>вонь
Зачем ты заставляешь меня постить гарантированные сообщения, я для тебя что ли сегодня свои калории ел?
Аноним 03/05/15 Вск 16:33:44 #439 №175031 
>>169138
Биткойны майнят небось.
Аноним 03/05/15 Вск 16:45:43 #440 №175039 
Что будет когда луна "улетит"?
Аноним 03/05/15 Вск 17:15:49 #441 №175049 
>>175039
Ну тебя точно не будет. и меня
Аноним 03/05/15 Вск 17:33:07 #442 №175062 
>>175039
Цикл приливов прекращается, климат начинает критические изменения, ось вращения ничего не стабилизирует, вымирание всех сложных видов жизни. Небольшое увеличение астероидной угрозы.

А вообще, такие вопросы годно гуглятся - всё уже расписано правда, иногда разные люди по-хардкору поясняют диаметрально противоположные точки зрения.
Пример: https://answers.yahoo.com/question/index?qid=20101218214137AA2Fo0g
Аноним 03/05/15 Вск 17:57:43 #443 №175089 
>>175062
С хуя ли из-за прил вов изменится климат? С хуя ли осьвращения должна вообще пойти по пизде? Шарик дохуя легче луны чтобы им ось стабилизировать? Луна имеетмассу 1.23% земной есличо. Да вообще никто нихуя не заметит.
Аноним 03/05/15 Вск 17:58:38 #444 №175091 
Еще один вопрос по Луне, не стукайте. Раз она к нам повернута всегда одной стороной, то откуда на ней кратеры? Ведь если что-то будет пролетать между нами то по идее ёбнуться должно на нас.
Аноним 03/05/15 Вск 18:04:37 #445 №175097 
>>175089
Приливы отвечают за облик гидросферы. Гидросфера прямо влияет на климат, рельеф и растительность, как и они все друг на дружку, а все вместе - на остальную жизнь, особенно на наиболее сложные её формы.

>Луна имеетмассу 1.23% земной есличо.
Мало что ли? Раскрути 1,23 % собственной массы на верёвке.
Аноним 03/05/15 Вск 18:12:30 #446 №175103 
>>175091
Смотря с какой скоростью. Да и вообще необязательно, траектория может быть самой причудливой.
Аноним 03/05/15 Вск 18:39:52 #447 №175114 
>>175091
>Диаметр земли - 12к
>орбита луны 300к
Не вижу проблемы попасть по луне, кроме ее малого размера. Но во время поздней тяжелой бомбардировки это не играло роли, пространство было засрано в край.
Аноним 03/05/15 Вск 18:44:47 #448 №175116 
14306678874520.png
Привет спейсач. У меня вопрос, насчет Space X Falcon 9. Все уже насмотрелись видео с эпичной посадкой и крушением первой ступени в вертикальном режиме.
https://www.youtube.com/watch?v=fs-XI58YeQc
https://www.youtube.com/watch?v=QyaUjOCxOX0
Но в чем смысл вертикальной посадки? Это имело бы смысл на планетах без атмосферы и с низкой гравитацией, где ошибки бы было проще исправить, да и снижение было бы не таким быстрым/жестким. На земле и на планетах с атмосферой же было бы логичнее использовать парашюты, или же в комбинации с той же вертикальной посадкой и надувными баллонами для амортизации. Единственный смысл в управляемой посадке я вижу в том чтобы корректировать снижение на подлете к земле, а затем - открывать парашюты в километре над землей.
Аноним 03/05/15 Вск 18:55:26 #449 №175122 
>>175039
Она не улетит, вот в чем дело. После того, как вращение Земли сильно замедлится, Луна снова начнет приближаться к Земле.
Аноним 03/05/15 Вск 19:09:11 #450 №175130 DELETED
>>175116
Платина.
На парашютах не получается достаточно мягкой посадки, плюс в баржу не попасть.
Нет, на землю не разрешают сажать
Аноним 03/05/15 Вск 20:05:41 #451 №175147 
>>175130
Следующая посадка как раз на землю, вообще.

1. Обкатка технологии.
2. Приземлиться на парашютах достаточно точно таки не получится энивей. При этом саму посадку можно смягчить двигателями, да.
3. Надо добавлять парашюты, которые весят дохуя, в верхней части, кстати. Упрочнять ступень, чтобы их не вырвало с мясцом. Потом перепаковывать их, тащить ракету на космодром. Проще нести с собой больше топлива (тем более, что оно энивей потребуется при коррекции снижения/посадке) и пару раз разбить ступень, тестируя софт.
Аноним 04/05/15 Пнд 14:13:44 #452 №175498 
Если вес P = mg, то выходит что масса 70 килограммового человека на земле равняется 7кг в космосе?
Аноним 04/05/15 Пнд 14:17:18 #453 №175503 
>>175498
Масса ≠ Вес
Аноним 04/05/15 Пнд 15:03:58 #454 №175526 
>>175498
И же == нулю на орбите.
Аноним 04/05/15 Пнд 15:25:33 #455 №175535 
14307423331970.png
>>175498
Во-первых, в космосе гравитация присутствует так же, как и на земле. Если подняться на высоту хоть геостационарной орбиты, но при этом не достичь нужной скорости движения (первой космической) - вес вполне будет присутствовать, только немного изменится g. Во-вторых, если ты движешься вокруг Земли с первой космической скоростью - это значит, что центростремительное ускорение равно -g. То есть, по модулю как g, но направлено в другую сторону. И получается, что P = mg - mg = 0. Невесомость.
Из твоей логики следует, что ты считаешь g в космосе равным 1. Это какая-то хуита.
Аноним 04/05/15 Пнд 18:39:18 #456 №175658 
Видно-ли Вояджеры в телескоп? Оптический, радио, ИК и проч.
Аноним 04/05/15 Пнд 18:45:02 #457 №175663 
>>175658
Нет. В радио и видимом диапазоне слишком большая дистанция для такого маленького обьекта, в ик излучает осне слабо.
Аноним 04/05/15 Пнд 19:51:00 #458 №175698 
14307582607620.jpg
>>175658
>>175663
На VLBA сам аппарат не виден, но его радиосигнал - да.
Аноним 04/05/15 Пнд 19:56:03 #459 №175702 
>>175698
Да вояджер-1 до сих пор передаёт данные о солнечном ветре м магнитном поле. Вояджер-2 вроде тоже до сих пор жив. Вот что случается когда не слушают зелёных пидоров и ставят нормальный источник питания.
Аноним 04/05/15 Пнд 20:01:36 #460 №175706 
Аноны, а как бы развивалось освоение космоса если бы США и СССР не мерялись пиструнами? Мы бы никогда не побывали на луне? Не запустили станции у марсу и Венере? Неужели меряние ракетами - наше все?
Аноним 04/05/15 Пнд 20:04:17 #461 №175713 
>>175702
>Вот что случается когда не слушают зелёных пидоров и ставят нормальный источник питания.
Стронция и цезия этому господину.
Аноним 04/05/15 Пнд 20:08:09 #462 №175718 
>>175663
А всякие пидоры в т.ч. в спейсаче мне рассказывают, что в космосе ничего нельзя спрятать и всё видно. Пиздят?
>>175698
Ну сигнал его радиостанции? Ну это не то, про что я спрашивал. Нахуй бы вояджер был бы нужен, если бы его радиосигнал не было бы слышно.
Аноним 04/05/15 Пнд 20:08:30 #463 №175719 
14307593105090.jpg
>>175713
Спасибо
Аноним 04/05/15 Пнд 20:10:21 #464 №175722 
>>175718
У вояджера нет движков. Потому и не видно.
Аноним 04/05/15 Пнд 20:10:24 #465 №175723 
>>175706
>Мы бы никогда не побывали на луне?
Скорее всего да, потому что дорого.
>Не запустили станции у марсу и Венере?
Скорее всего запускали бы, потому что не так дорого, алсо уже давно нет космической гонки, а АМС как летали, так и летают.
Аноним 04/05/15 Пнд 20:14:09 #466 №175729 
14307596497320.jpg
>>175718
Есть предел разрешающей способности. Вот например Плутон и Харон снятые Хабблом. Это блядь здоровенные тела вот так только снять можно. Что уже говорить о хуитке которая находится дальше них.
Аноним 04/05/15 Пнд 20:14:51 #467 №175731 
>>175729
Сорри, здесь Плутон только
Аноним 04/05/15 Пнд 20:19:55 #468 №175738 
>>175718
>что в космосе ничего нельзя спрятать и всё видно. Пиздят?
Теоретически не пиздят, практически очень трудно детектить объекты, которые слабо излучают. Если хуитка не движется, холодная, маленькая, покрашеная в черное и не передает радиосигналов, то задетектить ее очень сложно.
Аноним 05/05/15 Втр 12:06:13 #469 №175921 
Бамп. Поясните, почему в космосе нельзя передвигаться использую дизельный двигатель?
Аноним 05/05/15 Втр 12:11:54 #470 №175923 
>>175921
внутреннего сгорания? ты мудак? к чему ты собрался прикладывать механическую силу в вакууме?
Аноним 05/05/15 Втр 12:20:01 #471 №175926 
14308176017950.jpg
>>175921
Только если прилепить к нему гребной винт, и плыть по эфирным волнам.
Аноним 05/05/15 Втр 12:47:30 #472 №175930 
14308192503310.jpg
>>175926
Я конечно не знаю, но по-моему движители "Упоротого" совершенно не приспособлены для этой среды.
Аноним 05/05/15 Втр 12:56:16 #473 №175935 
>>175921
Можно, на тяге, создаваемой выхлопными газами.
Аноним 05/05/15 Втр 13:13:14 #474 №175937 
>>175921
>>175935
Можно на тяге, создаваемой выбрасываемыми назад дизельными двигателями.
Аноним 05/05/15 Втр 13:19:26 #475 №175940 
>>175937
Тоже вариант.
Аноним 05/05/15 Втр 13:30:14 #476 №175944 
Если космонавт в КА в космосе. оттолкнется от стенки и всей массой ударит в противоположную, корабль получит ускорение?
Аноним 05/05/15 Втр 13:41:06 #477 №175947 
>>175944
Только если у космонавта в руках EM-драйв
Нет, потому что закон сохранения импульса: импульс, который космонавт придаст кораблю при столкновении, будет компенсирован импульсом, который он придаст ему, отталкиваясь от стенки.
Аноним 05/05/15 Втр 13:41:28 #478 №175948 
>>175944
В общей сумме - нит.

А поколебать станцию способно. Потому и хотели и хотят запилить свободнолетящий компаньон для МКС (ОКА-Т) - топают космонавтики, особо чувствительным экспериментам идеальность микрогравитации нарушают.
Аноним 05/05/15 Втр 13:48:46 #479 №175949 
А если комсонавт прикрепит резинку от трусов к стенке, растянет ее и запустит груз или себя в ту стенку где прикреплена резинка?
Аноним 05/05/15 Втр 13:52:37 #480 №175950 
>>175940
И кпд будет крайне мал.
Аноним 05/05/15 Втр 14:12:19 #481 №175958 
>>175949
Закон сохранения таким образом не наебешь, успокойся.
Аноним 05/05/15 Втр 14:13:39 #482 №175959 
>>175944
в первом приближении нет как сказал этот анон>>175947, во втором приближении кратковременное возмущение орибты как сказал этот анон>>175948
но цимес в том что все эти темы с неоднородностью гравитационного поля и приливных сил оказываются гораздо забавней чем это могло показаться на первый взгляд
если он обдолбается соответсвующими таблетками то может иметь шанс через миллион лет допрыгаться и догнать луну
Аноним 05/05/15 Втр 15:16:35 #483 №175983 
14308281950810.jpg
>>171876
>>171881
за твои три квадратных км, я не поленился посчитал, но сразу совет не бери панели, бери зеркало и фокусируй, хоть на тех же панелях, но лучше на стрилинге - у него кпд по идее должен быть в этих условиях гдето под 90-95% (теор)
Аноним 05/05/15 Втр 15:59:48 #484 №175990 
14308307883230.jpg
>>175949
Если резинка оче крепкая, корабль тонкий, а космонавт - маленький и твердый, то космонавт проделав собою в стенке корабля сопло с рваными краями, придаст кораблю ускорение.

Кроме того корабль некоторое время таки будет получать ускорение, пока в нем не закончится своя атмосфера
Аноним 05/05/15 Втр 16:05:37 #485 №175991 
>>175948
> хотят запилить свободнолетящий компаньон
Что мешает запейлить такой копаньон внурти мкс?
Аноним 05/05/15 Втр 16:22:09 #486 №175993 
>>175990
А дырка в стене-то зачем, мамкин физик?
Аноним 05/05/15 Втр 18:53:10 #487 №176034 
>>175983
Что за место ха
Аноним 05/05/15 Втр 18:54:00 #488 №176035 
>>175990
Допустим стена из обеднённого урана.
Аноним 05/05/15 Втр 19:40:53 #489 №176050 
>>175991
Как ты себе это представляешь? Об стенки будет ебашить, воздухом телепать. А тут летает по соседней орбите и время от времени докается к станции.

Хотя сейчас с этой катавасией уже нихуя не понятно, что с этой Окой будут делать.
Аноним 05/05/15 Втр 21:50:06 #490 №176083 
14308518064060.jpg
>>176034
Your mom's pookan.
Аноним 05/05/15 Втр 22:09:46 #491 №176098 
Поясните в чем сложность отправить человека на марс. Ведь давно умеем стыковать йобы на орбите, можно сделать корабль на сколь угодно тонн натаскать прогрессами. И пиздец, МКС шмагли а два-три сраных блока для миссии на марс не можем. Чому?
Аноним 05/05/15 Втр 22:10:54 #492 №176099 
>>176098
>протонами
Фикс
Аноним 05/05/15 Втр 22:19:47 #493 №176104 
>>176098
Платиновый ответ на платиновый вопрос.
Нет автономности по ремонту.
Нет автономности по СЖО.
Недостаточно исследован медицинский вопрос длительного пребывания в изоляции, невесомости и радиации.
Нет ясности с защитой от радиации.
Мелкоракетами собирать дорого.
Дельты надо дохуя, это не Луна.
Нет смысла воткнуть флаг и бросить, надо лететь долгосрочно.
Нет стимула вообще туда лететь.
Список можно продолжать долго.
Аноним 05/05/15 Втр 22:26:10 #494 №176114 
>>176098
>можно сделать корабль на сколь угодно тонн натаскать протонами.
Нельзя, сто раз говорили. Протоны и даже Энергии с Сатурнами и SLS не подходят для сборки пассивных конструкций, они лишь доставляют на опорную орбиту. Чтобы пристыковать железяку на орбите к другой железяке, нужно какое-то подобие автономного грузовика, который доставит железяку, пристыкует, отшвартуется и утопится/сядет. Такого грузовика после шаттла нет и не планируется. По сути, собирать кучу железяк на орбите нормально научились только начиная с МКС, раньше были стыковки независимых кораблей (Мир например, или российские модули МКС).
Аноним 05/05/15 Втр 22:28:24 #495 №176115 
>>176098
Дорога к марксу не устлана розовыми лепестками, не прикрыта магнитосферой земли как земная орбита, поэтому считается, что все кто отправятся туда в бочках типа апполонов - прилетят либо "горячими" либо вообще мертвыми. Отсюда необходимость биологической защиты, как следствие - возрастание массы, возрастание либо необходимой мощности движков и соответственно, необходимого запаса топлива, или увеличение сроков полета и соответственно необходимых запасов еды и опять таки требований к биозащите.
Проблема ж в том облет марса человеком, без посадки - не слишком впечатляющее достижение, чтобы под это дело можно было бы вырубить достаточные средства, да еще в условиях возрастания междугородной напряженности.
Аноним 05/05/15 Втр 22:32:37 #496 №176117 
>>176114
Кстати, как сборка была реализована в марсианской части Constellation?
Аноним 05/05/15 Втр 22:35:29 #497 №176118 
14308545293770.jpg
>>176114
> отшвартуется и утопится/сядет.
а если не утопится, перелетит на орбиту схоронения, а потом когда надо, спустися обратно?
Аноним 05/05/15 Втр 22:40:21 #498 №176121 
>>176104
Все решаемо я думаю, просто никто не хочет тратить денежки. Единственные кто полетит лет через пятьдесят - китайцы.
Аноним 05/05/15 Втр 22:42:56 #499 №176126 
>>176118
Можно и так, но я бы не стал к станции гонять ядерную йобу.

>>176121
Дело не в денежках. Не выйдет лететь к марсу прямо сейчас, по той же причине как и нельзя заплатить стопицот триллионов нефти и стать интелом за месяц.
Аноним 05/05/15 Втр 22:44:07 #500 №176130 
>>176118
Надолго не хватит все равно. И 20-30тонные модули поседеешь такой штукой таскать
Аноним 05/05/15 Втр 22:51:36 #501 №176136 
14308554961400.jpg
>>176118
Алсо это все же разгонный блок, а не корабль-робот типа Прогресса или Дракона. Он годится гонять между орбитами, НОО-ГСО, например. А не стыковать. Чтобы стыковать нужны маневровые движки, а это неизбежный гидразин, максимум водород (если учитывать проектируемые йобы). Бывают всякие z-free режимы, но маневровые все равно необходимы.
Аноним 05/05/15 Втр 23:32:08 #502 №176163 
>>176034
Место икс, в какойто степени отвечающее требованиям спрашивавшего, но вообще тоже надо было диапазано нарисовать
так как 100 ватт на 3 кв километра это чтото в духе 15 световых суток вместо 3ёх , разница как бы есть
Аноним 05/05/15 Втр 23:42:02 #503 №176171 
>>176136
ПН пусть сама маневрирует, буксир токо орбиту занять, но так да у него должен быть качественно лучший УИ иначе смысл практически теряется. в принципе каконить буксир с ядерным движком мог бы нахапывать рабочее тело на низкой орбите. в этом случае это был бы четкий вин
Аноним 06/05/15 Срд 00:09:02 #504 №176178 
>>176171
>ПН пусть сама маневрирует
Спутники умеют, спутники таскать она годится (хоть маржины и очень малы, даже окупаемость под вопросом, мне вообще кажется её делают просто по приколу). А вот железки и обитаемые бочки не умеют. Для этого нужны СУ и ДУ, опять получаем автономный робокорабль с гидразином.
Аноним 06/05/15 Срд 01:08:40 #505 №176191 
Я все понимаю, но почему нет треда по EmDrive'у?
Аноним 06/05/15 Срд 01:34:18 #506 №176201 
>>176191
В глаза ебешься? На нулевой висит
https://2ch.hk/spc/res/120283.html
Аноним 06/05/15 Срд 01:44:34 #507 №176205 
>>176191
>Я все понимаю, но почему нет треда по EmDrive'у
Долбоебы сделали ДВА треда по ЭмДрайву, один висит на нулевой, но все равно придет какой-нибудь особо тупорылый имбецил и спросит ПОЧЕМУ НЕТ ТРЕДА.
Аноним 06/05/15 Срд 08:35:39 #508 №176269 
Почему космонавтам не сделают магнитные башмаки что они ходили нормально?
Аноним 06/05/15 Срд 08:53:28 #509 №176272 
>>176269
По чему ходить-то?
Аноним 06/05/15 Срд 08:58:12 #510 №176273 
>>176117
Каждый модуль был самостоятелен.
Аноним 06/05/15 Срд 09:06:32 #511 №176274 
>>176272
По поверхности. Постелить металлические решётки например
Аноним 06/05/15 Срд 09:22:15 #512 №176276 
>>176274
>По поверхности. Постелить металлические решётки например
По какой? Забыл, что они в бочках живут?
Аноним 06/05/15 Срд 09:47:05 #513 №176284 
>>176276
Ну а бочка разве не металлическая?
Аноним 06/05/15 Срд 10:05:31 #514 №176288 
На какой скорости обьект упадёт на солнце?
Аноним 06/05/15 Срд 10:19:18 #515 №176292 
1. Правда ли что на планетах в ядре галактики всегда светло от множества ярких звёзд?
2. Если допустить существование чд в ядре галактики, возможно ли что она может оказывать заметное действие на биологические обьекты?
3. При каких условиях планета может оторваться от звезды и отправиться блуждать среди звёзд?
4. Можно ли передавать информацию быстрее скорости света, скукоживая пространство наподобие принципа Альбукеркекубрере.
Аноним 06/05/15 Срд 11:01:29 #516 №176300 
Где тут тред обсуждения инопланетян? Какие-то шары ульбукерке обсуждают, воздействия какие-то, вы аутисты что-ли?
Аноним 06/05/15 Срд 11:06:41 #517 №176302 
14308996015020.jpg
14308996015041.png
Согласно аналемме Земля переодически то "обгоняет", то "уступает" Солнцу?
Аноним 06/05/15 Срд 11:15:02 #518 №176308 
https://2ch.hk/spc/res/139511.html ну хоть один тред нашел в потоке говна, что вы тут обсуждаете, пиздос, не раздел, а загон для дебилов со своими псевдонаучными охуительными высерами.
Аноним 06/05/15 Срд 11:26:49 #519 №176312 
>>176300
>>176308
-> /b/
Аноним 06/05/15 Срд 11:28:29 #520 №176313 
>>176312
Хуй соси, даун.
Аноним 06/05/15 Срд 11:30:00 #521 №176314 
>>176292
1. Хуй знает, если там внутре как снаружи - куча пыли, то не факт.
2. Ясен хуй, поток жесткого рентгеновского излучения это мягко выражаясь НЕПРИЯТНО для любого биологического объекта, например твоей мамки
3. Гравитационный захват мимопролетающей звездой, например.
4. Нет/никто не знает
Аноним 06/05/15 Срд 12:18:00 #522 №176326 
Поясните по хардкору, можно ли будет в ближайшем будущем наваливать боком среди звёзд под Элвиса, употребляя джин и раскуривая дудку?
Аноним 06/05/15 Срд 13:04:57 #523 №176342 
>>176302
Орбита у планеты не круговая, а эллиптическая, соответственно и скорость не постоянная.
Аноним 06/05/15 Срд 13:15:58 #524 №176349 
>>176326
Ты и сейчас это можешь делать, просто вспомни что мы и так среди звезд летаем на шарике.
ПЕРЕКАТ Аноним 06/05/15 Срд 13:25:14 #525 №176351 
https://2ch.hk/spc/res/176350.html
https://2ch.pm/spc/res/176350.html
Аноним 06/05/15 Срд 13:31:04 #526 №176357 
14309082645360.jpg
14309082645381.gif
14309082646912.gif
>>176342
Уточню. Гифрелейтед предположим, что там показано движение Солнца вокруг центра масс СС. За счет ускорения и замедления движения Земли оная то летит немного впереди, беря светило на обгон, то затормаживаясь и отставая, так? За счет эклиптики СС относительно диска галактики?
Аноним 06/05/15 Срд 13:43:48 #527 №176366 
>>176357
http://masterok.livejournal.com/1982988.html
Вот тут доходчиво.
Аноним 06/05/15 Срд 14:00:41 #528 №176373 
>>176366
Не то, но все равно благодарю.
а ответ получил- да, за счет эклиптики самой СС
Аноним 06/05/15 Срд 14:04:43 #529 №176376 
14309102839980.jpg
>>167732
Что мешает нам директору какого-нибудь Яндекса или Avito свою космическую программу заиметь?
Меня аж трясло всего когда я узнал что Blue Origin это проект Амазона.
Не получится так что нашу космическую программу будут с автоВАЗом сравнивать через пару десятилетий? Такой-же никому не нужный дотационный металлолом?
Аноним 06/05/15 Срд 14:07:05 #530 №176379 
>>176376
Так трясло что перекат не заметил.
Аноним 06/05/15 Срд 14:09:16 #531 №176381 
>>176376
Потому что мерседес важнее Космоса
Аноним 06/05/15 Срд 15:01:47 #532 №176394 
>>176269
Получится что то вроде этого
https://www.youtube.com/watch?feature=player_detailpage&v=wr-7zcUuS90#t=132
только без падений.
Аноним 06/05/15 Срд 15:23:40 #533 №176398 
>>176376
>Не получится так что нашу космическую программу будут с автоВАЗом сравнивать через пару десятилетий?
Ты хотел сказать, с десяток лет назад начали сравнивать? Нихуя нового она так и не родила, даже за пределы орбиты практически не выбралась. Мир построили по инерции от Советов, МКС помогали строить, пару аппаратов интересных запустили на орбиту. На этом все.
Аноним 13/05/15 Срд 12:18:16 #534 №178836 
>>176394
Не получится. Станция не настолько ЙОБА, чтобы иметь сантиметровой толщины стальные листы на полу и по всей обшивке. Там в основном тонкий легкий металл, типа жести или фольги, везде, где не требуется держать нагрузки, т.е. только силовой каркас более-менее толстый, остальное фольга.
К фольге магниты не магнитятся - слишком тонкая, чтобы замкнуть поле магнита. Тем более к алюминиевой.
Короче говоря магнитам там не за что цепляться - МКС это не боевой крейсер с метровой стальной броней.
 #535 №172279span>  DELETED
>>172057
И у меня. Но в хорошем смысле.
comments powered by Disqus

Отзывы и предложения